Está en la página 1de 50

1 A Dentist did pulpotomy of primary molar and then after few moths he did

stainless steel crown. What band he gets to claim.


Band 1
Band 2
Band 3
Band 4
No Claim

2After doing mesio-angular impaction of lower third molar, which suture


material is
best used to close mucoperiosteal flap
3-0 Silk cutting needle
3-0 Vicryl cutting needle
Catgut

3 Patient has TIA (Transient ischemic attack three months ago, you have
planned
extraction and patient is on aspirin. What is the best way to proceed?
Delay the extraction for three months
Go ahead with extraction, following appropriate local
measures
Refer to Oral surgery
Stop aspirin

4 Patient has an over jet of above 9mm which UDA band it will
be
Band 1
Band 2
Band 3
Band 4
Band 5

5 Over jet of 4.5 mm criteria which UDA band it will


be
Band 1
Band 2
Band 3
Band 4
Band 5

6 Autoclave temperatures (at what temperature the sterilization is


achieved)
121 degrees centigrade for 15 minutes
121 degrees centigrade for 3 minutes
131 degrees centigrade for 15 minutes

7 A patient with irritable bowel syndrome what do you


see.
Mucosal blistering
Mucosal tags
Herpitiform ulcers

8 Patient had pain in the chest after prolonged dental session, with an extraction procedure, what is the best medicine to give
GTN
ASPIRIN

9 Patient is insulin dependent diabetic, complains of faint after prolonged


dental
session, what is best to give
Oral Glucose
IV Glucagon
Insulin
1M Glucagon

10 Patient is waiting in waiting room and collapse and faints. Patient is cold, clammy, but pulse is good. What do
you give?
IM Glucagon
Oral Glucose
GTN
Hydrocortison
e
Diazepam
d.
c.
f.
e.

11 Patient is suffering myocardial infarction in dental chair, which position is best for
patient
Upright sitting position
Laying flat
Recovery position
Lay the pt flat and raising legs

12 A 90-year old gentleman presented to clinic who is edentulous and has dentures upper and lower full is-years old.
Denture bit
uncomfortable, tooth structures little bit worn out, freeway space 2-4mm, polished surfaces satisfactory and occlusal
wear
minimal. How would you proceed?
Copy dentures
Hard reline
Soft reline
Construction of new dentures

13 When do you, do not extract impacted 8's


Extraction of 8's recommended by
orthodontist
Extraction of 8's recommended in patient undergoing renal transplant -immunosuppresent
47years old with moderate periodontitis
Second permanent molar undergoing internal resorption due to third molar

14 Non dental origin, No clinical or radiological findings. Patient complains of pain maxillary sinus and ear deafness,
unilateral ulcer
on face and forehead
Herpes Simplex
Herpes Zoster
Paramyxo virus
Orthomyxo virus
Ep-stein bar virus

15 How do you check maxillary


plane
Fox-bite-plane
OCcluding the upper and lower
denture

16 What sort of matrix is best for restoring disto-occlusal restoration


of 7?
Sectional matrix
Auto matrix
Tofflomeir matrix

17 How do you get incisal guidance for setting incisal


teeth?
Study casts
Unsupported/supported labial fullness

18 Burning mouth syndrome which part of the oral cavity is more


painful
Tongue
Lichenoid reaction seen associated with
Atenolol
Ramipril

19 GTN in Angina
Relaxes heart muscle
Decreases blood
Pressure
Increase positive ionotropic
charge
Dilation of arterioles and
venules

20 had trauma - Nasal- midface area and has watery discharge and blood discharge from nose
Orbital fracture
Zygomatic fracture
Lefort-l
Lefort-ll

21 Patient with only mid-root fracture of front tooth, what is apropriate splinting time?

1 week

2 week

4 week

3 months
h.
g.
j.
i.
22 Patient with lower four central incisors with alveolar fracture, you splinted and immobilized successfully, and then
what is the
next thing to do.
Soft diet and review

23 Patient had trauma with minimal mobility of front teeth, no symptoms or pain or anything, what is the best
approach
Splinting one week
Splinting two weeks
Splinting 4 weeks
Splinting 6-8 weeks
Soft diet and review ~

24 Patient had trauma 8 days ago, upper central incisor palatal luxation mild, not interfering with occlusion but tooth non
vital.
What do you do?
Pulp extirpation +CaoH2
dressing
Reposition
Reposition +Splinting ~

25 Best material for using vital pulpotomy


..,Ferric sulphate
Formocresol

26 Patient had trauma on 14 (vital pulp), only thin buccal cusp remaining, best treatment
option
MOD-Onlay
inlay
Direct composite MODL
Extract and options for replacement

2718 months of age, what is fluoride supplement for child living in 0.25 ppm fluoridated water?
0.25mg
0.5 mg
1 mg

28 Six-year-old - Non fluoridated water, what is the fluoride supplement


dose
0.25 mg
0.5 mg
1 mg

29 Six-year-old child with fluoridated water of 0.6ppm, what is the fluoride supplement dose
given
0.25 mg
0.5 mg
1 mg

30 3-year old with fluoride concentration of 0.3 ppm fluoridated water, what is the fluoride supplement dose
given
0.25 mg
0.5mg

31 Duraphat in high caries risk group children is applied


every
2 months
4monttls ~
6 months
8 months
One-yearly

32 Reason for failure of eruption of lower permanent


molar
Canine palatally place
Crowding in premolar area
Failure of eruption of central
incisor
Absence of showing lateral
o. incisor
k.
m.
l.
n.

32 Reason for failure of eruption of lower permanent


molar
Canine palatally place
Crowding in premolar area .
Failure of eruption of central
incisor
Absence of showing lateral
incisor
33 Failure of normal path of
eruption
Abnormal position of crypt
Early loss of deciduous
teeth
Supernumerary tooth

34 Cervical line and dark lines on Central incisors, molars and incisal edges, most possible reason
could be
Ameologensis
Dentinogenesis
Childhood illness
Hypoplasia

35 An edentulous patient presents with ulcer in retro molar pad area first line of management would be
Adjusting the denture
Incisional biopsy
Periapical xray

36 How should be the floor of dental


surgery?
impervious
Absorbable

37 Possible infection that would occur from needle stick injury in a person who is appropriately or properly
immunized
Hepatitis B
Hepatitis C
Hepatitis A
HIV

39 Got a call from mother of a 3-year old child, who had trauma and avulsed the front tooth. What instruction you will give to

Surgery as soon as possible with tooth


Place tooth in milk and attend surgery as soon as possible
,Place in the salt water and attend surgery as soon as possible
Place the tooth in its position back and attend surgery as soon as possible

40 A 3-year-old boy avulsed 61, and mother re-implanted it in its socket and clot is holding the primary tooth. IOPA
shows
permanent tooth present, what would you do
Leave it and review
Splinting for one week
Remove and space maintenance

41 A child has grossly broken down upper and lower molars, carious tooth, which x-ray would you
consider
IOPA
Bitewings_
Vertical
Bitewings
DPT

42 Patient has painful symptoms at the time of eating and also has ear pain, what x-ray would you consider. (The option of
occlusal
x-rays was not given in the choices)
OPG
Oblique Lateral -
Lateral chepalograph
Postero-anterior view

43 A dental practitioner can be


1-Radiologyadviser+physicst+operator
2-Superviso r + adviser+physicst
3-practioner+refer+operatOL

4-Practioner + physicst +supervisor


5-Physicst + supervisor + 0perator
p.
q.

44 Risk of cancer in dental Radiography (IOPA)


1- 1:20,000
2- 1:200,000
3- 1:2,000,000
4- 1:20,000,000

45 Which cells are affected by Bisphosphonates?


Mesencymal cells
Undifferentiated cells
Oesteoclasts

47 A diabetic patient complains of pain on a root canal treated tooth. Root canal is done 2 years ago. On examination the
restoration on the tooth seems leaking. Reasons for pain and failure
No proper obturation
Medical condition
Lack of coronal seal
Inadequate biomechanical preparation

48 Best success of root canal is achieved by


Straight canals
Straight line access
Widening of aperture of root canals
Proper irrigation all the times under presence of rubber dam

49 When preparing the Apical Zone, the use of the files sequentially from apex to backwards (lower the size of instrument)
- what
is the best distance to achieve good apical area preparation
0.5m
m
1.0m
m

l.5mm
2.5m
m
3.5m 50 Prolonged use of corticosteroids in adults may result in or can
m cause
Alopecia
Insulin - diabetes
Libidos
Thinning of
skin
Less arousal
51 Best material to use in the root caries and buccal caries in elderly patient.
Hybrid
Composite
Amalgam
Silicate
cement
Zinc
phosphate
r.
52 A child patient is brought to the dental clinic, parents complain of child having trauma and has fallen down, hurting his
teeth.
You suspect NAI's. Which is the most appropriate finding which would give suspension of NAI'S?
Red eye
Neck bruise
Child agitated and distressed
Injuries to shin and knee areas
53 What is the bpe score if gingival recession of 2mm and band completely
disappears
1
2
3
4

54 What is the bpe score if Gingival recession 2mm +5.5mm


1
2
3
4
55 On examination of four teeth in a sexant except for L4 has no bleeding on probing. What is the BPE
SCORE?
O
1
2
3
4
56 Patient has damaged infra-orbital nerve - orbital floor fracture, patient presents to you after two hours later. What

symptoms
you would find, choose most appropriate one
Peri-orbital
edema
Loss
57 of visual
what you would do
1-Dismissal
acuity
Loss of sensation
2-Grievance or paresthesia over cheek
procedure
3-Informal discussion
4-Suspension and investigatin
5-Written warning

Options are
a-Dental assistant want to explore future carrier opportunitie
b-Below options that needs to be matched
c-Dental assistant suspected of the theft
d-Repeated complaints of letters from patients regarding dental assistant
e-Dental staff or person failure to respond repeatedly even after giving many
f-A staff member repeatedly re-heard many times comments from colieagues

58 For retention of post which is not important


Diameter
Size
Luting cement
Type of post

59 RCT treated tooth, post placed, which is the best option to restore the tooth.
Nayyar
technique used, good tooth structure 50%
Gold crown
Gold inlay
PJC
Gold Onlay

60 The movement of tongue on protrusion is affected to lateral side -which nerve is involved. Patient recently had
surgery and
radiation close to submandibular gland
Lingual
Glossopharyngeal
Hypoglossal
Chordatympani
61 When patient complaints about a treatment and unhappy about the procedure and gives a complaint, in how many
days you
have to acknowledge the complaint
1 day
2 days
3 days
10 days
20 days

62 Patient has cardiac arrhythmias on warffarin and yesterday had INR of 3, today needs extraction. What do you do?
Differ the extractions
Consult phvsician
Alter the dose of warfarin
Do the extraction and control post operative bleeding

63 Chose the correct option


1-General anesthetics and day surgery
2-General anesthetics and one day inpatient
3-Local anesthetics and oral sedation
4-Local anesthetics and inhalation sedation
5-Loca I anesthetics

Options

a-Child 3-years-old, removal of all deciduous


primarymolars
b-12- years-old very anxious patient, removal of all 5's
c-Extraction of fully erupted molar in 4-month-old pregnant
lady
d-Child for extractions who is medically compromised

64 Secondary impressions for edentulous patient close fit tray with hyper gap reflex. Which material you use for
taking an
impression
Alginate
Silicone rubber based
material
Plaster of Paris
Zinc oxide eugenol

65 Palatally impacted upper maxillary canines. How would you know? Whether they are impacted palatally or buccally and
which of
the following x-rays are best option
Paralleling technique
Bisecting technique
Bitewings
OPG
Taking two x-rays, with paralleling technique

66 Defective horizontal angulations while taking bitewings


results in?
Overlapping of contact points
Dark radiograph
Light radiograph

67 Which of the following antifungal medication interact with warfarin and enhances its
effect?
Ketacanazole
Micanazole_
Flucanazole
Amphotrocin B

68 A patient came after two days of extraction, complains he had pain for two days, examination showed localized
swelling, no
lymphadenopathy. What is the first line of treatment?
Possible dry socket -irrigation and placement of sedative
medicament
Irrigation - curettage of socket, antibiotics and analgesics
Irrigation and antibiotics
Possible root piece or bone piece take IOPA x-ray and analgesics
69 Patient getting treatment done under IV Sedation - suddenly not responding has decreased blood pressure and
decreased
breathing
Oversedation
Asthma

70 Patient is having epileptic seizures continuously for 10 minutes in dental surgery - first line of
management is
Place in recovery position
Give diazepam
Oxygen
Intranasal or buccal midolazam

71 Sudden unilateral onset of facial swelling and pain over parotid area, which is not yet
fluctuant
Bacterial saliadenitis
Salivary duct infection
Saliorrhea
Viral siliadenitis

72 What do you see in irritable bowel


syndrome?
Mucosal blistering
Herpitiform ulcers
Mucosal tags
Macular patches on buccal mucosa

73 A fifteen year old patient asks for bleaching of teeth. What is the first line of management?
Discuss options with both patient and parent
Discuss option with both patient and parent, once you have obtained consent from
patient
Discuss only with patient
Discuss only with parent
-

74 A child avulsed permanent tooth came with her aunt, what will you
do?
Do not do anything
Do not do treatment as aunt is not legal guardian of the child
Do not do treatment as child's mother is not there to give consent
Re-implant and splint it with aunt's consent (I have gone for this option, because treatment can be some times done, if it
is in
child's best interest)

75 Best ways to reduce radiation for patient who is going to have IOPA X-
rays
Lead apron
Paralleling technique
Rectangular collimation
D-Film

76 Match the option


Apical periodontitis
Lateral apical abscess
Crack tooth syndrome
Dentine hypersensitivity
Hyperemic pulp
Reversible pulpitis
Options
Intermittent pain from molar which is having huge restoration
leaking
Prolonged intense pain for several days
Acute pain which decreases 10-15 minutes after the stimulus is
removed
Intermittent pain on biting
Intermittent pain with hot things
77 Match the options
Simple cantilever conventional
Simple cantilever minimal
Spring cantilever conventional
Spring cantilever minimal
Fixed-fixed conventional
Fixed-fixed minimal
Fixed movable

Options to match
Missing upper right central 11 and restored 21
Lateral incisor has restorations, pointed canines
Canines pointed, lateral incisors spaces,
diastema
46 and 44 present 45 missing, 44 minimally tilted, 46 heavily filled both are tipped 46mesial and 44 distal

78 What is the best cement used to cement minimally done inlay restoration (MOD)
GIC
ZNO
Zinc phosphate
Zinc poly carboxylate
Resin cement

79 What sort of matrix is best for restoring disto-occlusal restoration of


7?
Sectional matrix
Auto matrix

80 Patient had trauma with minimal mobility of front teeth, no symptoms or pain or anything, what is the best approach
Splinting one week -v
Splinting two weeks
Splinting 4 weeks

81Patient had trauma 8 days ago, upper central incisor palatal luxation mild, not interfering with occlusion but tooth non
vital.
What do you do?
~ Pulp extirpation +CaoH2
dressing
Reposition
Reposition and splinting

82 Patient had trauma on 14 (vital pulp), only thin buccal cusp remaining, best treatment
option
MOD-On lay
Inlay
Composite
83-Deciduous tooth had intrusion 61, what is most likely to occur for permanent
tooth
Impaction
Uneruption
Hypoplasia
Dilaceration

84-Splinting time for Avulsion


2 weeks
1 week
7-10 days
85 Got a call from mother of a 3-year old child, who had trauma and avulsed the front tooth. What instruction you will give
to
mother
Place tooth in milk and attend surgery as soon as possible
Place in the salt water and attend surgery as soon as
possible
Place the tooth in its position back and attend surgery as soon as possible

86 A 3-year-old boy avulsed 61, and mother re-implanted it in its socket and clot is holding the primary tooth. IOPA
shows
permanent tooth present, what would you do
Leave it and review:
Splinting for one week
Remove and space maintainence

87 A child has grossly broken down upper and lower molars, carious tooth, which x-ray would you
consider
IOPA
Bitewings
Vertical Bitewings
88 When patient complaints about a treatment and unhappy about the procedure and gives a complaint, in how many days
you
have to acknowledge the complaint
1 day
3 days
10 days
20 days
89 Patient has cardiac arrhythmias on warfarin and yesterday had INR of 3, today needs extraction. What do
you do?
Differ the extractions
Alter the dose of warfarin
Do the extraction and control post operative bleeding

90 Match the questions to the options


l)General anesthetics and day surgery
2)General anesthetics and one day
inpatient
3)Local anesthetics and oral sedation
4)Local anesthetics and inhalation
sedation
5)Local anesthetics

OPTIONS
a)Child 3-years-old, removal of all deciduous primary molars
b)12- years-old very anxious patient, removal of all 5's:
c)Extraction of fully erupted molar in 4-month-old pregnant
lady
d)Child for extractions who is medically compromised

91 Secondary impressions for edentulous patient close fit tray with hyper gap reflex. Which material you use for
taking an
impression
Alginate
Plaster of Paris
Zinc oxide eugenol

92 Palatally impacted upper maxillary canines. How would you know? Whether they are impacted palatally or buccally and
which of
the following x-rays are best option _
Take 2 Paralleling technique radiographs
Bisecting technique
Bitewings
OPG

94 Which of the following antifungal medication interact with warfarin and enhances its
effect?
Ketacanazole
Micanazole
Amphotrocin B

95 A patient came after two days of extraction, complains he had pain for two days, examination showed localized
swelling, no
lymphadenopathy. What is the first line of treatment?
Irrigation - curettage of socket, antibiotics and analgesics
Irrigation and antibiotics
Possible root piece or bone piece take IOPA x-ray and analgesics
Irrigation and debridement

97 Patient is having epileptic seizures continuously for 10 minutes in dental surgery - first line of
management is
Place in recovery position
Midazolam
Give diazepam
Oxygen

98 Sudden unilateral onset of facial swelling and pain over parotid area, which is not yet
fluctuant
Salivary duct infection
Sialadenitis
Saliorrhea

99 What do you see in irritable bowel


syndrome?
Mucosal blistering
Mucosal tags
Macular patches

100 A fifteen year old patient asks for bleaching of teeth. What is the first line of
management?
Discuss options with both patient and parent
Discuss only with
patient
Discuss only with
parent

101 A child avulsed permanent tooth came with her aunt, what will you
do?
Reimplant with aunts consent
Do not do anything
Do not do treatment as aunt is not legal guardian of the child
Do not do treatment as child's mother is not there to give consent

102 Best ways to reduce radiation for patient who is going to have IOPA
X-rays
Lead apron
Paralleling technique
D-Film
Rectangulr collimation

103 EMQ based question to match the


options
Apical periodontitis
Lateral apical abscess
Crack tooth syndrome
Dentine hypersensitivity
Hyperemic pulp
Reversible pulpitis

Options
Intermittent pain from molar which is having huge restoration
leaking?
Prolonged intense pain for several days?
Acute pain which decreases 10-15 minutes after the stimulus is
removed?
Intermittent pain on biting?
Intermittent pain with hot things?

104 Match the


options
a.tetracycline stain
b.mild flourosis
s. hypo plastic pit
t. Food staining extrinsic stain

Treatment
options
Crown
Microabration followed by
veneer
Bleeching(chair side}
Whiting tooth paste

106 Which test is the most reliable test to indicate the presence of active
hepatitis
H bs(su rface }antigen
Hbc(core)antigen
Hbe antigen
Antibody to HBs antigen

107 First line treatment to needle stick


injury
Wash the area under running water
Sera pe the a rea
Refer to infectious disease specialist

108 what is legal amount (unit}of alcohol intake for


men
18
21
24

109 Medical condition of pt who reserved a seat in a dinner meating for his dead
wife
Mania
Anxiety
Depression
Schizphrenia
110 Lady in 40s got TMJpain for about few months, she got divorced recently what is the first line
treatment
a.amitrptyline
b.physiotherapy
c.occlusal splint

111 which epithelial cell that responsible for the


attachments?
9-what type of cells that proliferate in later stager of her
life?
a.Odontoblast
b.cemtoblast
c. rest of malazess
e.undifferentiated mesynchymal cells

112 BLS no of
compressions
40:1
30:1
30:2

113 Patient fit and healthy then he got cold and clammy then he lost his
consciousness
Syncope
Epilepsy
Hypoglycaemia

114 Drug of choice to acute asthma


Salbutamol
Steroids

11S The term abrasion best describes:


A.Loss of substance by chemical agent
(erosion)
B.Loss of substance by external agent
(abrasion)
C.Loss of substance by the movement of tooth against tooth (attrition)
D.The rapid loss of substance that is seen in the movement of porcelain crowns aganist natural teeth (attrition)

116 Some days after preparation and filling of a shallow class I amalgam cavity the patient complains of pain on biting. You
would:
A.Perform vitality
test
B.Replace filling
C.Check for premature contacts
D.Remove all occlusal contacts from this filling
E.Tell the patient to wait 2-4 weeks, the pain will go away

117 In a composite filling, the matrix band is


for
A.Help shaping and contouring the filling
B.Prevent material to be pushed under the gingival margin

120 In a class 11.2 malocclusion, which bridge design would be contraindicated for a missing lateral upper incisor?
A.Cantilever bridge
B.Maryland bridge

121 Which is the best cantilever bridge design for missing maxillary canine?
Abutment on
A.Both premolars
B.Lateral and central
incisor
C.Lateral incisor
D.First premolar

122 A 9 years-old child who has sustained a fracture of a maxillary permanent central incisor in which 2 mm of the pulp is
exposed,
presents for treatment 30 minutes after injury. Which of the following should be considered?
A.Remove 1-2 mm of the pulp tissue surface, place calcium hydroxide and fill with resin
B.Remove 1-2 mm of the pulp tissue surface and cover with ledermix
C.Place calcium hydroxide directly on the exposed pulp
D.Pulpotomy using formocresol
E.Pulpectomy and immediate root filling

123 In a flouridated toothpaste with 0.304% sodium fluoride the amount of flouride
ions is
A.400 ppm
B.l000 ppm
C.1500
ppm
D.4000
ppm

124 The most important diatary habit for caries


development is
A.Amount of sugar intake
B.Frequency of sugar intake
C.Form of sugar intake

125 The normal unstimulated salivary flow


rate is
A.0.02 ml/min
B.0.2 ml/min
C. 2 ml/min

126 Titanium is used in


dentistry
A.ln a very pure form in
implants
B.ln an alloy with aluminium in casting for crowns and
bridges
C.ln an alloy with nickel in orthodontic wires
D.A and B
E.A, Band C

127 What is the reason for a tooth to develop pulpitis several years after setting of a full veneer gold
crown?
A. Bacterial microleakage

128 How is the regeneration process after damage by injury to odontoblasts


working?
A.Proliferation of the remaining odontoblasts
B.Differentiation from fibroblasts
C.Regeneration from undifferentiated mesenchyme I
cells
D.Histodifferentiation from ectodermal cells
E.Differentiation from the inflammation cells

129 In construction of full dentures, what does the term "too low vertical dimension" refer to?
A.A situation in which there is too much interocclusal space between upper and lower artificial teeth when the mandible is
in rest
position.

130 Reversible pulpitis is characterized by


A.Pain lasts longer on hot or cold stimulus than
normally
B.Patient can't localize pain
C.Wili have periapical involvement in radiograph

131 Irreversible pulpitis is characterized


by
A.There is often a history of spontanous
pain
B.Sudden throbbing pain
C.Pain can't be localised when it reaches the periapical area
D.There is pain which lingers for a short duration after romoval of stimulus

132 What are the symptoms of internal


resorption?
A.very painful
B.Symptom-free or only mild pain

133 What kind of root fracture in a tooth has the best prognosis? A fracture
at the
A.Apical third
B.Coronal third
C.Middle third
D.Vertical fracture

135 which of these disease that caused by positive lacto


bacillus
Syphilis,TB
Angular cheilitis

136 Child with sore throatand feeling unwell,he got macular rash on his
cheek?
a.measles
b.chicken pox
c.herpes simplex
137 Diabetic pt with abscess complain of failure of root canal therapy and during the examination the filling was leaking?
What the
reason
a.lack of coronal
seal
b.lack of apical
seal
c. Pt medical condition

138 what is the most common site that affected by burning mouth
syndrome?
Tongue
Palate
Cheek

139 Pt worried of getting cancer what the advice that should u


give?
Regular check up
5 fruit/veg a day

140 Pt got recently metallic taste sensation after taking oral


medication?
a.burning mouth syndrome
b.oral dysthesia
c.oral dysgeusia

141 Reasons for crowding


A early loss of deciduous teeth
b.delayed eruption of lower first
molar
c primary failure of eruption
143 Reason for delayed eruption of lower first
molar
a ea rly loss of decid uous teeth
b.delayed eruption of lower first molar
c primary failure of eruption

142 Reason for palatal position of upper


canine
a early loss of deciduous teeth
b.delayed eruption of lower first molar
c primary failure of eruption
d abnormal position of crypt

143 A patient in your dental chair shows chest pain, weak pulse and dysponea, what is your initial management,
A. Administer nitro-glycerine and keep the patient up seated
B. Put the patient in supine position
C. Wait until the symptoms go away

144 Developer was contaminated with other chemical and was not mixed properly. What is the effect on the X-ray film?
A. Too dark film
B. Light film
C. Foggy

145 Branchial Cleft cyst is located


A. In front of the neck
B. On anterior border of the Sternocleidomastoid muscle
C. Shows when swallowing

146 What is the approximate unstimulated salivary flow rate,


A. 2 ml/min
B.0.2 ml/min
c. 0.02 ml/min
D. 20 ml/min

147 Some hours after the extraction of a lower molar the patient complains of prolonged post operation bleeding and
pain, how
would manage this,
A. Prescribe analgesics and ask the patient to follow a strict oral hygiene
B. Administer 5% Marcaine Local Anaesthetic, prescribe analgesics and pack the socket with alvogyl
C. Administer 5% Marcaine Local Anaesthetic, suture the socket and prescribe analgesics
D. Suture and give pressure packs
148 A Gracey curette is characterized by
A. The blade and the shank form a 902angle
B. Can be used on both sides
C. Can be used on any tooth surface
D. It is specific for each surface ofthe tooth

149 A patient with no positive history came along for scaling. The moment you pick up your anterior scaler you punch your
finger,
what should you do?
A. Complete the procedure as if nothing has happened
B. Check dentist's blood for Hepatitis B antibody HBsAb
C. Check dentist's blood for Hepatitis B antigen HBsAg
D. Check patient's blood for Hepatitis B antibody HBsAb and HIV antigen HIVAg
E. Check patient's blood for Hepatitis B antigen HBsAg and HIV antibody HIVAb
F. Dentist should go and take a HBsAb vaccine
~
150 A patient on the dental chair has cardiac arrest. What is INCORRECT,
A. Observing the vital signs and check that the air way is clear is at high importance
B. Expired air has 15% 02 only, and cardiac compressions achieve 30-40% of cardiac output
C. Intermittent positive pressure at the rate of 40/min will reduce the chances of ~erebral hypoxia
D. Intermittent positive pressure is better than mouth to mouth when it has been given at the same rate.
E. You check the pulse and respiration of the patient before starting any resuscitation

151 Treatment for small carious cavity in co operative


child
Pulp capping
Pulpotomy
Pulpectomy

152 Treatment for pinpoint exposure in symptomless


tooth
Pulp capping
Pulpotomy
Pulpectomy

154 Pt with labially impacted upper canine, how does u know


clinically?
Swelling on the labial area
Swelling on the palatal side

155 First line management of dry


socket
a.irrigation.x ray.metrondiazole 200
b.irrigation xray metro diazole 400
c.irrigation and x ray only

156 First line management of pt with palatal dietary


erosion
a.crown.,
b.palatal veener
c.direct composite

157 Preferred bridges for missing


UR2
A Minmal prepartion adhesive
bridge
B Conventional adhesive bridge
C Fixed-fixed bridge
D Minimal preparation fixed
fixed
E Simple cantilever
F Spring cantilever

158 Preferred bridges for missing


ULl
A Minmal prepartion adhesive
bridge
B Conventional adhesive bridge
C Fixed-fixed bridge
D Minimal preparation fixed
fixed
E Simple cantilever
F Spring cantilever

158 Preferred bridges for missing


UR3
A Minmal prepartion adhesive
bridge
B Conventional adhesive
bridge
C Fixed-fixed bridge
D Minimal preparation fixed
fixed
E Simple cantilever
F Spring cantilever

159 Preferred bridge for missing 1st


molar
A Minmal prepartion adhesive bridge
B Conventional adhesive bridge
C Fixed-fixed bridge
D Minimal preparation fixed
fixed
E Simple cantilever
F Spring cantilever

160 Preferred bridge for missing


1stpremolar
A Minmal prepartion adhesive bridge
B Conventional adhesive bridge
C Fixed-fixed bridge
D Minimal preparation fixed
fixed
E Simple cantilever
F Spring cantilever

161 Preferred local anesthesia option for a normal fit


pt?
A Lidocaine + adrenaline
B Prilocaine+felypressine
C Bupivcaine
D Benzocaine
E Articaine

162 LA with prolonged duration of


action?
A Lidocaine + adrenaline
B Prilocaine+felypressine
C Bupivcaine
D Benzocaine
E Articaine

163 Preferred option of la for pt with heart


disease?
A Lidocaine + adrenaline
B Prilocaine+felypressine
C Bupivcaine
D Benzocaine
E Articaine

164 Flavoured topical


anesthesia?
A Lidocaine + adrenaline
B Prilocaine+felypressine
C Bupivcaine
D Benzocaine
E Articaine

165 which of these jobs dentist can do it?


A radiotion protection supervisor+phsices
b.radiation protection
supervisor+practioner
c. practio ne r +refe re r +0 pe rato r

166 Pt on warfarin, what is the accepted inr to do simple


extraction
A2
B3
C4
D5

167 which of this pt has risk of dry


socket
a.smoker
b.pt on warfarin
c.eldery pt

168 what is the most reasonable indication for third molar


removal
A First episode of pericorinitis
B Second episode
C Periapical infection with erupted wisdom tooth

169 what the feature of the carpet of surgery?

170 Dentist did root canal treatment previously to child pt and now he do a crown for the same pt what band the dentist
should
claim?
Band 1
Band 2
Band 3
No calim

171 what ur advice to childs mum whos her child s teeth knocked
out?
a.Reimplant it
b.keep it in the milk and come as soon as possible
correct
c.keep it in the water

172 Child accidentally consumed 5mg/kg flouridewhat the first line


treatment?
Give child salty drink
Give child sugary drink

173 What is the most correct flap design in


apecictomy?
Apical re position
Apex is broader tha n the
base
Base is broader than the
apex
Apex and base are equal

174 Common irrigation solution for


endodontices?
A sodium hypochlorite
B Saline
C Chlorhexidine

175 What will develop after prophylaxis?


A.Aceliular pellicle is formed immediately
after
B.Celiular pellicle is formed immediately
after
C.Acellelar pellicle is formed after 48 hours
D.Cellular pellicle is formed after 48 hours

176 What is the purpose of making a record of protrusive relation and what function does it serve after it is made?
A. To register the condylar path and to adjust the inclination of the incisal guidance.
B. To aid in determining the freeway space and to adjust the inclination of the incisal guidance.
C.To register the condylar path and to adjust the condylar guides of the articulator so that they are equivalent to the
condylar
paths of the patie nt.
D. To aid in establishing the occlusal vertical dimension and to adjust the condylar guides of the articulator so that
they are
equivalent to the condylar paths of the patient.

177 A 50 years-old patient presents with pain from time to time on light cervical abrasions. What is your first management
to help
patient in preventing pain in the future?
A.Change diatary habits
B.Change brushing habits
C.GIC fillings

178 In planning and construction of a cast metal partial denture the study cast
A. facilitates the construction of custom trays
B. minimizes the need for articulating
C. provides only limited information about inter ridge distance, which is best assessed clinically
D. can be used as a working cast when duplicating facilities are not available

179 What are the artificial teeth in removable dentures made


of?
A.Porcelain
B.Cross-linked methyl-
methacrylate
C.Ethyl-methacrylate
D.Acrylic

180 Following extraction of the molar teeth


A. The ridge height is lost more from the maxilla than from the mandible
B. The maxillary ridge will get more bone lost from the palatal aspect than the buccal
C. The mandibular arch is relatively narrower than the maxillary arch
D. Compared with the pre-resorption state, the mandibular ridge will lose more bone from the lingual aspect than the
buccal one.

181 Which anatomical landmark is important to include in impressions for lower full
dentures?
A.Mylohyoid ridge
B.Lower incisive papilla

182 Wrought metal is to be,


A. Marble
B. Quenched
C. Has undergone cold treatment during processing

183 Which ofthe following is ONE indication for indirect pulp


capping?
A.When further excavation would lead to pulp exposure
B.Excavation of a very deep caries

184 How does fluoridation work in theory?


A. Fluoride ions are integrated by changing Hydoxylapatite to Fluorapatite

185 A major difference between light cured and chemical cured composite is that during setting or in function the light
cured
materials tend to:
A. Seal the margins better and completely
B. Exhibit less wear on time
C. Undergo greater colour change
D. Shrink more rapidly
E. Posses greater fracture toughness

186 What consideration is important in deciding if a bridge for upper missing incisors should be made in pontic design
or with
gingiva imitation?
A.Wishes of patient
B.Bone resorption in edentoulos span
187 The most common cause of porosity in porcelain jacket crowns is,'
A. Moisture contamination
B. Excessive firing temperature
C. Failure to anneal the platinum matrix
D. Excessive condensation of the porcelain
E. Inadequate condensation ofthe porcelain

188 How should the occlusion in partial removable dentures be


designed?
A.Artificial teeth should be out of occlusion
B.Artificial teeth should not interfere with the incisal guidance established by the remainig natural teeth.

189 The minimal labial tooth reduction for satisfactory aesthetics with porcelain fused to metal
crown is,
A.1mm
B. The full thickness of enamel
C.1.5 mm
D.2.5mm
E. One third of the dentine thickness

190 In removable partial denture, the principle of an indirect retainer is to:


A. Stabilise against lateral movement
B. Prevent settling of major connectors
C. Restrict tissue movement at the distal extension base of the partial denture
D. Minimise movement of the base away from the supporting tissue

191 When a removable partial denture is terminally seated the retentive clasps tips should:
A. Apply retentive force into the body of the teeth
B. Exert no force
C. Be invisible
D. Resist torque through the long axis of the teeth

192 Glass lonomer Cement sets because of,


A. Acid-Base reaction
B. Addition polymerisation reaction
C. Growth of glass crystals
D. Slip plane locking
E. Solvent evaporation

193 The reflex in gagging patients is caused by:


A. Trigeminal nerve
B. Glossopharyngeal
C. Facial nerve
D. Recu rrent la ryngea I

194 The use of nickel chromium in base plate should be judiciously considered because:
A.A significant number of females are allergic to nickel .,
B. A significant number of females are allergic to chromium
C. A significant number of males are allergic to nickel

195 Which of the following liquids is not suitable for prolonged immersion of cobalt chrome partial
dentures:
A. Alkaline peroxidase
r>. B. Sodium hypochlorite
C. Soap solutions
D. Water

196 In complete dentures, cheek biting is most likely a result of:


A. Reduced Overjet of posterior teeth
B. To high vertical dimension
C. Teeth have large cusp inclines

197 The most common cause of fracture at the isthmus of a class II dental amalgam
restoration is:
A.lnsufficient condesation
B.Fracture line developing from pulpal-axial angle of the
cavity
C.Underconturing of the isthmus area
D.Moisture contamination of the amalgam during
placement
E.lnadequate bulk of amalgam at pulpo-axialline angle

198 Why do you overpack amalgam


fillings?
A. To remove excess mercury
B To prevent microleakage

199 What is CORRECT in regard to the periodontal surface area in maxillary teeth:
A. central incisor> first premolar> second premolar
B. Canine> first premolar> central incisor
C. Canine> lateral incisor> second premolar
D. Canine> central incisor> first premolar

200 When restoring with composite resins, why do we do the cavo-surface bevelling:
A.Aesthetic
B.To open enamel rods for acid
attack
C.To smooth preparation
D.A and B
E.AII of the above

201 A well constructed complete denture:


A. Needs little maintenance
B. Needs less than a week for adjustment and total success
C. Has adverse effects and decreases taste sensations

202 On examination of a composite restoration you find a dark attain:


A. Replace the composite
B. Repair with unfilled resin
C. Apply topical fluoride at the margin

203 A patient complains of sensitivity, on examination you find a composite filling restoring a good cavity preparation
without any
secondary caries; what is your next step:
A. Extirpate the pulp that is obviously inflamed
B. Place ZOE dressing to sedate the pulp
C. Ask patient to come back in six months
D. Repeat restoration

204 What is the best way to cement a Maryland


bridge,
A.GIC
B. Resin cement
C. Composite resin
D. Zinc Phosphate cement
E. Oxide Zinc and eugenol

205 The ideal length of a post in the fabrication of crown and core of endodontica!ly treated tooth is:
A.2/3 of tooth
B.the tooth
length
C.1.5 times that of the
crown
D.Yz root length
E.The length of the crown

206 While you finish a class I cavity, the enamel is sound but you notice a thin brown line ln the dentine and on the dentino-
enamel
junction, what is your response,
A. You leave it and complete the final restoration
B. You extend your preparation and clean it
C. You apply a cover of varnish

207 Dental caries ofthe proximal surfaces usually starts at,


A. Somewhere between the ridge and the contact area
B. Just gingival to contact areas
C. Just above the gingival margin
D. At the contact point

208 The Frankfort plane is defined by which anatomocallandmarks,


A. Porion, orbitale
B. Sella, orbitale
C. Nasion, Tragus

209 The biting load of a denture base to the gingival tissues compared to teeth are,
A. Ten times more
B. Ten times less
C. Equal

210 The difference between normal stone and the dye stone is,
A. In the size and shape of the particles
B. The mixing

211 The advantage of the silicone in soft relining material over hard plastic acrylic materials is,
A. Capability to flow
B. Prevents the colonization of Candida albicans
C. Resilient in long run
D. Better bond strength

212 A female patient comes to you complaining of persistent pain in a heavily restored central incisor; you suspect
irreversible
pulpitis and you have been told that she is in transit leaving by plane next day. Your treatment will be,
A. Remove filling and place a sedative dressing
B. Pulpectomy and Ledermix dressing
C. Pulpectomy and calcium hydroxide dressing
D. Prescribe analgesics and systemic antibiotic

213 The flexibility of the retentive clasp arm does not depend on:
A. Length of the arm
B. The cross section shape
C. The material used
D. Degree of taper
E. The exerted force

214 Following calcium hydroxide pulpotomy, the dentist would expect dentine bridge to form at,
A. The exact level of amputation
B. Level somewhere below the amputation
C. Half way between amputation and apex
D. At the apical region of the tooth

215 In the construction of a full veneer gold crown, future recession of gingival tissue can be prevented or at least
minimised by,
A. Extension of the crown 1 mm under the gingival crevice
B. Reproduction of normal tooth incline in the gingival one third of the crown
C. Slight over contouring of the tooth in the gingival one fifth of the crown
D. Slight under contouring of the tooth in the gingival one fifth of the crown

216 What is correct in regard to high copper amalgam,


A. Reacts and strengthens the amalgam by its dispersion properties
B. Reacts to form copper-tin phase thereby eliminating the tin-mercury phase
C. Reacts to form copper-silver phase thereby eliminating the silver mercury phase
D. Reacts and strengthens the amalgam by its grain diffusion

/">. 217 The removable partial denture requires relining,what would be the most appropriate action,
A. take an impression by asking the patient to occlude on it
B. Provide equal space between denture and gingival tissues.
C. Make sure the framework and retainers are seated in place before taking impression
..:'

218 Stiffness of material is measured by


A. Proportional unit
B. Modulus of elasticity
C. Stress/ stra i n
D. Ultimate tensile strength

219 Two central incisors on a radiograph are showing with what looks like eye drop radiolucency. You decided to start
endodontic
treatment on these teeth but when you tried to open access to the root canal you find clearly closed orifices with what look
like
secondary dentine. What is your initial management?
A. Leave as it is and start a permanent restoration.
B. Start systemic antibiotic
C. Try to ream and file canals

220 After the initial development stage and in the absence of pathology, the size of the pulp chamber has been reduced by,
A. Deposition of primary dentine
B. Deposition of secondary dentine
C. Reparative dentine
D. Pulp fibrosis
E. Deposition of reparative dentine

223 Denture stomatitis is commonly associated with,


A. The continuous wearing of removable orthodontic appliances in otherwise healthy patient
B. The proliferation of hypertrophic tissue at the denture periphery
C. The overgrowth of some constituents of oral normal microflora
D. Allergy to denture base material

224 The light emitted by the polymerization lamp has to be checked from time to time. The meter used for this only
measures light
in the range of:
A. 100-199 nm
B. 200-299 nm
C. 300-399 nm
D. 400-499 nm

225 Which is correct in regard to shade selection of crowns:


A. It should be selected before starting preparation
B. Chroma is the lightness/darkness of colours
C. Value is the colour itself
D. Hue is the concentration of colours

226 Where would you expect to find the mylohyoid muscle in relation to the periphery of a full lower
denture:
A.Mandibular buccal in the midline
B.Mandibular lingual in the first premolar
area
C.Mandibular lingual in the midline
D.Mandibular disto buccal area

227 After reimplantation of an avulsed tooth the prognosis may be poor


because of
A.External resorption
B.lnternal resorption

2282.2 mg of NaF contains how many mg of fluoride


ions?
A.O.5mg
B. 1.0 mg
C.1.5m
g
D.10mg

229 How would you treat denture


stomatitis?
A.Nystatin
B.Tell the patient to leave the denture out for some days

230 What is true about third molar surgery?


A.Swelling is maximum after 24 - 48 hours
B.Mental paraesthesia indicates careless
technique

2311n anaesthizing a 70 kg healthy man with Lignocaine 2% with 1:100,000 vasoconstnctor.what is


correct?
A.The toxic threshold is 22 ml
B.2.2 ml is the maximum you can give in one session
C.Lignocaine has the same anaesthizing capacity as
Benzocain
D.Lignoscain is 5 times less potent than Bupivacain
E.Lignocain is more dangerous in Hypothyreodism than Bupivacain

232 A suddenly swollen upper lip that lasts for 48 hours or more is most
likely
A.Haemangioma
B.Agioneurotic oedema
C.Mucocele
D.Cyst

233 What is the most important factor to reduce radiation in dental radiographs?
A. Speed of film
B. Collimation
C. Filtration
D. Cone shape and length
E. Use of lead apron

234 What is the best way for a permanent decline in caries of a


population?
A.Change diatary habits
B.Topic and water fluoridation
C.Awareness of dental health matters
D.Better tooth brushing

235 Loss of sensation/paraesthesia in the lower lip may be produced by,


A. Bell's palsy
B. Traumatic bone cyst
C. Trigeminal neuralgia
D. Osteomyelitis
E. Ludwig's angina

236 In anxoius and psychologically stressed patients gingivitis is often more severe
because of
A.Stress causes histamine and serotonine release
B.5tress causes catecholamine and corticosteroid release
c.Stressed people neglect their oral hygiene

237 A retained lower primary incisor causes the permanent


incisor to
A.erupt buccally
B.erupt lingually
C.ankylose

238 What is the least probable consequence in thumb-


sucking?
A.Reclining of lower incisors
B.Protrusion of upper incisors
C.Formation of deep palate with big overbite

239 Ankylosis of teeth is often found after changes in the continuity of the occlusal plane. These changes are
caused by
A.Overeruption of opposing teeth
B.Localised growth inhibition of the alveolar
bone
C.Sinking of ankylised tooth into the bone

240 What is not important in obduration materials for primary


teeth?
A.Good apical seal
B.Radioopacity
C.Antibacterial
D.Resorbable

241 What is your first consideration in the treatment of dry socket?


r>; A.Prevention of
osteomyelitis
B.Pain relief

242 A patient presents with pain in the upper left segment. On inspection you f~nd a localized alveolar abcess distal 27.
What will be
you management? -
A.Drainage
B.Extraction of tooth

243 What does not help in establishing the caries risk in


children?
A.History of caries
B.Lactobacilius count
C.Dietary habits
D.Brushing habits
E.Genetic predisposition
244 Opioid drugs are similar to which endogenous substances?
A. Bradykinins
B. Peptides
C. Prostaglandins
D. Serotonins
E. Enkephalins

245 Which of the following have a tendency to recur if not treated correctly?
A. Giant cell granuloma
B. Lipoma
C. Fibrous epulis
D. Haematoma
E. Pulp polyps

246 When no radiation shield is available, the operator should stand out of the primary x-ray beam at a distance from the
patient's
head of at LEAST:
A. 0.5 metres
B. 1 metre
C.1.5
metres
D.2 metres
E. 3 metres

247 Which of the following is a typical consequence of dental crowding, assuming no primary tooth has been lost
prematurely?
A. Overlapping of lower incisors
B. Palatal displacement of upper canines
C. Impaction of 15 and 25 between first premolars and first molars
D. Mesial tipping of 16 and 26
E. Rotation of 16 and 26

248 What is the dominant microflora in acute necrotic ulcerative gingivitis (ANUG)?
A. Spirochaetes and fusobacterium SP
B. Spirochaetes and eikenella corrodes
C. Polymorphs and lymphocytes
D. Actinobacillus actinomycetes comitans oral capnocytophaga
E. Porphyromonas gingivalis and prevotella intermedia

249 Which of the following is true regarding gingivosis (Desquamative gingivitis)


A. It is caused by hormononal imbalance
B. Is seen only at or after menopause
C. Is frequently caused by lichen planus
D. Is a variant pregnancy gingivitis
E. Is related to nutritional disturbance

250 What are the points that determine the facial line in cephalometric points
A. Nasion, pronasale, pogonion.
B. Sella, nasion, pogonion

251 A 10 year old boy presents with small greyish white lesion surrounded by a red halos on the soft palate and tonsillar
pillars,
small vesicles are found. He has fever and pain in the ear and won't eat. The MOST probable diagnosis is?
A.Herpangina
f\ B.Measles
C.Primary herpetic stomatitis

252 A 12 years-old child presents with symptoms of widespread gingivitis with !lleeding and general malaise for several
weeks. How
would you manage this patient?
A. Prescribe Metronidazole 100mg
B. Locally debride, give oral hygiene instruction and prescribe H202 mouth wash.
C. Give a prophylaxis with ultra sonic scaling
D. Refer for haematological screening
E. Advise for bed rest with supportive and palliative treatment

253 What is the effect of office dental prophylaxis of regular six month intervals on children's oral health?
A. Reduced caries incidence by approximately 30%
B. Provide a long term improvement in oral hygiene
C. Provide a short term improvement in oral hygiene
D. Prevent gingivitis
E. Reduce the need for patient cooperation

254 What is the most frequent cause of pain which occurs several days after obturation?
A. Entrapped Bacteria in the periapical region
B. Underfilling the root canal system
C. Overfilled root canal

255 A diabetic patient with moist skin, moist mouth and weak pulse; what would you do:
A. Give glucose orally
B. Administer 02
C. Administer adrenaline
D. Inject insulin

256 A patient has developed a sever chest pain and difficulties in breathing while in the dental chair. Your initial
response is:
A. Administer glycerine trinitrate and monitor patient in upright position
B. Patient has an acute episode of angina as demonstrated by curve in ECG
C. No treatment is required until confirmed as MI by ECG
D. Patient has myocardial infarction as confirmed by ECG

257 In the case of malignant melanoma occurring intraorally, which of the following is true:
A. Uncommon on the palate
B. Should not be biopsied, as this will increase metasis
C. The 5 years survival rate is 20%
D. The incidence of oral melanoma is the same as those on the skin
E. Commonly occurs intra orally

258 What is NOT TRUE in relation to the use of diazepam for sedation?
A. Patient commonly complain of post operative headache
B. An acceptable level of anxiolytic action is obtained when the drug is given one hour preoperatively
C. There is a profound amnesic action and no side effects
D. Active metabolites can give a level of sedation up to 8 hours post operatively
E. Can be used safely for children

259 Patient on anti-coagulant therapy requires an extraction to be performed. Which of the following is NOT true:
A. Minor leedings bleeding can be reduced somehow by using tranexamic acid
B. Prothrombin value above 2.5 is required to perform extraction
C. It takes up to 12 hours for Vitamin K reverse effects of warfarin
D. Heparin can be administered sub-cutaneous and acts rapidly

260 A physician refers a nine year old boy to you to confirm diagnosis. The boy has a fever of 40°C and coughing. When
you focus
your light into his eyes he turns away. Intra-orally there are white spots surrounded by red margins. The disease and
lesions are:
A. Measles and the spots are Koplik's spots
B. AHGS vesicles
C. Rubella and the spots are Fordyce's spots

261 What is true in TMJ dysfunction


therapy?
A.Should be treated surgically
B.Appliances that raise the bite usually relief the symptoms and are used prior to any surgery

262 What is true regarding pregnancy


gingivitis?
A.lt is due to increased gingival
microcirculation
B.Elevated oestrogen and gestagen levels are directly responsible .;
C.Hormonal changes cause the growth of anaerobic bacteria (Prevotella interrnedia)

2635 mm probing depth means:


A.Patient has periodontitis
B.Probe is 5 mm beyond gingival margin
C.Probe is 5 mm beyond dentino-enamel junction

264 A middle aged woman gives a history of intermittent unilateral pain in the sub mandibular region, most probable
cause is,
A.Calculus in the salivary duct resulting in sialolithiasis. .
B. Ranula
C. Cyst
D. Mucocele

265 By which of the following mechanism reduces Aspirin pain:


A. It is anti inflammatory by the release of histamine
B. It blocks the cyclo-oxygenase pathway.

266 In minor oral surgery, what is TRUE in regard to antibiotics:


A. Amoxil satisfactorily covers the dental spectrum
B. Metronidazole and Amoxil have the same penetrating power
C. It is evident that it will reduce post operative swelling
D.There is convincing evidence that Prophylactic prescription of antibiotics will reduce postoperative
infections
E.Most oral infections get anaerobic after 2 to 3 days

267 A patient comes with a firm, painless swelling of lower lobe of parotid which has grown progressively for the past
year. He
complains of paresthesia for the past 2 weeks. This is most likely to be:
A. Pleomorphic adenoma
B. Carcinoma of the parotid
C. Lymphoma of parotid

268 What is true in treating a patient with secondary herpes simplex:


A. Acyclovir inhibits viral transcription when applied in the prodromal phase
B. Idoxuridine is better than acyclovir when applied topically
C. Antivirals are contraindicated in immuno-compromised patient
269 During extraction of a maxillary third molar the tuberosity is fractured; however, it remains in place attached
to the
mucoperiosteum. Which of the following procedures should be employed:
A. Remove the tuberosity and suture
B. Leave the tuberosity and stabilize if required
C. Remove the tuberosity and fill the defect with Gelfoam then suture.
D. If fractured tuberosity is greater than 2 ern, leave in place and suture

270 An incision biopsy of an ulcerated and intruded clinically suspicious lesion in a SO year old female reveals chronic
inflammation;
you would:
A. Inform the patient and her physician of your findings and instruct the patient to return in six months
B. Surgically excise the entire lesion since you know it is not malignant
C. Dismiss the patient with instructions for warm saline rinses and re-examination
D. Repeat the biopsy

271 Reducing the size of the focal spot will:


A. increase sharpness
B. increase density

272 The initial priority in treatment of horizontal fractures is:


A. Preservation of pulp
B. Immobilisation
C. Root canal treatment
D. Calcium hydroxide treatment

273 Which of the following has proven to be the MOST important in community preventive program:
A. Dental awareness of the community
B. Institution of oral hygiene measures
C. Water fluoridation

274 What effect has placing a sealant over pits and fissures on the progression of
caries?
l.Decreased new caries
2.lncreased new caries
3.Progression of exististing caries
4 No effect on existing caries

275 In advanced periodontitis with marked mobility, teeth may be splinted:


A.To improve comfort for the patient
B.splinting helps in transmitting the force to the adjucent teeth to reduce the load on the involved teeth

276 Swallowing will aid in the diagnosis of:


A. Branchial cyst
B. Thyroglossal duct cyst
C. Ranula
D. Retention cyst
F. Glossothyroid cyst

277 Which of the following will increase sharpness:


A. Larger focal spot
B. Smaller focal spot
C. Increase object-film distance
D. Decrease patient-source distance

278 In severe periodontitis the probe will eventually be:


A. prevented to go deeper by calculus
B. beyond connective tissues in the junctional epithelium
C. at the end of the junctional epithelium
D. Touching the middle of junctional epithelium
E. Touching sulculuar epithelium

279 A 58 year old male has been treated with radiation for carcinoma of tongue. The patient complains of pain
associated with
poor dentition. The dental management would be:
A. Immediate extraction of any poor teeth under local anaesthetic with antibiotic coverage
B. Segmental dental clearance and closure to eliminate problems
C. No dental treatment may be due to neuronic of neoplasms
D. Clearance of poor dentition followed by hyperbaric oxygen treatment plus a primary closure of wounds under
antibiotic
coverage
E. No extraction as radionecrosis is an important sequelae

280 Which of the following is NOT true about anticoagulation therapy?


A. INR of 3 is enough to start any extraction
B. Affects extrinsic system and increases prothrombin time
C. Heparin can be given subcutaneously and acts rapidly
D. It takes at least 12 hours for Vitamin K to reverse the effects of coumarin

2811 n a radiograph the roots of the upper teeth are too short because of:
A. Inadequate horizontal angulation
B. Too high vertical angulation
C. Too small vertical angulation

282 Characteristic of Squamous Cell Carcinoma of the tongue


A. more in white skinned people
B. more in alcohol drinking smoking males
C. associated with Plummer-Wilson-Syndrome

283 Characteristic of Squamous Cell Carcinoma of the lips


A. It reacts far simply to radiotherapy
B. metastizes mainly by blood
C. relatively rare in Australia

284 Which type of dentin is not formed due to pulp


pathology?
A.Reparative dentin
B.Secondary dentin
C.Primary dentin
D.Reaction dentin
E.Tertiary dentin

285 Which is not true in sickle cell anaemia?


A.Deformed cells with less oxygen transport
capacity
B.Higher infarction risk
C.Have wide bone marrow spaces with narrow trabeculae in the alveolar bone of oral
cavity
D.Resistant to malaria parasites
E.More common in mediterranean people

286 Normal prothrombin time and elevated partial thromboblastin time is seen
in
A.Factor VIII deficiancy (Haemophilia)
B.Thrombocytic pupura
C.Leukemia
D.Von Willebrand disease

287 Which of the following describes best a 9 years-old child permanent


dentition?
A. 1612 11 I 212226

464241 I 313236

B. 12 11 I 2122

4241 I 3132

288 What is the best reason to promote tooth brushing to the


public?
A.Less fissure caries
B.Less gingivitis
C.Gingival massage

289 What is untrue about diabetes?


A.Hypoglycaemia is more common than hyperglycaemia
B.lnsulin-dependend patients are of more concern than non insulin-
dependend
C.Adrenalin causes a decrease in the blood glucose level

290 Which of the following is a feature of Streptococcus


mutans?
A.lt does not require a special environment to grow
B.lt can be easily transported from one part of the oral cavity to
another
C.lt has the ability to restructure carbohydrates

291 What does the term "caries prevalence" mean?


A.The total number of carious areas affected and any present
caries
B.The individual risk for a patient to acquire caries
292 Which of the following are features of herpetic gingivostomatitis?
1. Irritability
2. Fever
3. Occurs in teenagers
4. Vesicles occur only on buccal mucosa and tongue

293 A patient in your dental chair suddenly becomes agitated with shallow breathing, full pulse and a blood pressure of
150/80.
You would
A.Give oxygen
B.Give insulin
C.Give glucose
D.Place patient in supine position

294 The principle clinical sign of active bruxism is:


A. Head and / or neck pain
B. Excessive tooth wear
C. Temporomandibular joint clicking
D. Sensitive teeth

295 An adult patient attends your practice complaining of pain-and swelling aSsOCiated with a previously restored
upper first
premolar tooth. The pain has been present for a number of days and is no longer responding to analgesics. His
dentition is
otherwise well maintained and his periodontal health is good.
What is the most appropriate approach to treatment?
A. Antibiotics and analgesics.
B. Extract the tooth
C.Carry out a pulpotomy and temporary
dressing.
D Drainage

296 A patient reports that his post crown has fallen out. This crown had been present for many years. You note that there
appears
to be a hairline vertical fracture of the root. The tooth is symptomless.
What is the most sensible approach to treatment?
A. Replace the post crown using a resin-reinforced glass ionomer material
B. Replace the post crown using a polycarboxylate cement
C.Replace the post crown using a dentine bonding agent and a resin-reinforced glass ionomer
material
D Replace the post crown using a resin composite luting agent
E. Arrange to extract the tooth

297 A patient says that he does not like the appearance of his previously root filled upper central incisor tooth. His
dentition is
otherwise well maintained and his periodontal health is good. The tooth appears to be darker than the adjacent teeth.
What is the most appropriate approach to treatment?
A. Provision of a post crown
B. Provision of an all ceramic crown
C.Provision of a metal bonded to ceramic
crown
D Non vital bleaching
E. Provision of a porcelain veneer

298 A 25 year old male attends for the first time complaining of sensitivity of a number of teeth. On examination, the
occlusal
surfaces of all the teeth are worn with obvious wear facets on the canines and premolars. Posterior amalgam
restorations are
proud of the surrounding tooth.
What would be the first stage management?
A. Take impressions for study models
B. Prescribe fluoride mouth rinse
C. Replace the amalgam restorations
D. Dietary analysis

299 A 35 year old male patient who admits to grinding his teeth at night has a number of wedge-shaped cervical
(Class V) lesions on
his upper premolar teeth. These are causing sensitivity and are approximately 3mm deep.
What is the correct management option?
A. Provide tooth brushing instruction and fluoride
B. Restore the lesions with compo mer
C. Restore the lesions with micro-filled composite
D. Restore the lesions with a hybrid composite
E. Restore the lesions with conventional glass-ionomer

300 A patient attends with pain of four days duration in a carious upper molar tooth. The pain is constant and
is not relieved by
paracetemol. Sleep has been disturbed by the pain. The tooth is tender to percussion and gives a positive
response to Ethyl
Chloride.
What is the most likely diagnosis?
A. Pericoronitis
B. Apical periodontitis
C. Marginal periodontitis
D. Reversible pulpitis

301 A 14 year old patient attends with a decayed and hypoplastic LL7. A radiograph shows the presence of an
unerupted LL8 and
the LL6 is sound.
What would be the most appropriate long-term treatment for this tooth?
A. Amalgam restoration
B. Antibiotics
D. Root canal therapy
E. Sedative dressing

302 A 30 yr-old patient attends complaining of pain from the lower left quadrant. Clinical examination
reveals a dentition with
generally good oral hygiene. There is no significant periodontal pocketing other:.than an isolated defect in
the region of the
furcation of lower left first molar which is non-mobile. The gingival tissue in thiS area appears erythematous
and slightly
hyperplastic with a purulent exudate on probing.
From the list below, which is the most appropriate next step?
A. Obtain a radiograph
B. Biopsy the gingival tissue
C. Remove the restoration
D. Vitality testing
E. Prescribe antibiotics

303 A 40 yr old patient had root-canal treatment to his upper first molar. This was performed 6 months ago
using contemporary
techniques under rubber dam and was crowned after completion of treatment. He attends complaining of
continued discomfort
from this tooth. Radiographic examination shows each of the three roots to be obturated with a well-
condensed filling to the full
working length though there is no evidence of in-fill of the periapical lesion when compared to the pre-op view.
What is the most
likely cause of the continued problem?
A. Extra-radicular infection
B. Contamination of canal(s) with E.faecalis
C. Uninstrumented canal
D. Ve rtica I root fra ctu re
E. Perio-endo problem

304 A 21 year old female presents for the first time to your practice. She is very upset with the appearance of her
upper left central
incisor. On examination you find healthy oral hard and soft tissues and excellent oral hygiene. On close
examination you can see
that the upper left central incisor is slightly greyer than the upper right central incisor and has a composite
restoration placed
palatally.
What is the most appropriate form of treatment given the information you have?
A. Bleaching with carbamide peroxide in custom formed trays of upper and lower arches
B. A bonded crown
C. A composite veneer
D. A porcelain veneer

305 EDTA (ethylene diamine tetra-acetate) has useful roles in certain situations in
clinical dentistry.
When would you use EDTA?
A. As a root end filling material
B.As a pulp
capping
agent
C As root
canal irrigant
D. As a mouthwash
E. As a dentine bonding agent

306 A patient presents with a history of clicking from their temporomandibular joint. This click occurs mid way through the
opening
cycle and is consistent. There is some pre-auricular pain and the lateral pterygoid muscle on the affected side is tender to
resisted
movement test. There is no trismus and the click is not present when the patient opens from an incisor edge to edge
relationship,
instead of her normal Class I occlusion. The patient would like treatment.
The most appropriate occlusal splint for this patient would be:
A. Stabilisation splint
B. Localised Occlusal Interference Splint
C. Bite Raiser
D. Soft Bite Guard
E. Anterior Repositioner Splint

307 A patient presents with a history of a post-crown having fallen out. The post-crown was originally placed fifteen years
ago and
had been successful up until four months ago since when it has come out and been recemented four times. At
recementation there
was no evidence of any caries. The patient had been a regular attender and not needed any restorative treatment for the
last eight
years.
Which of the following is the most likely cause for the failure of this crown?
A. The post was to narrow
B. The post was to short.
C. The root canal treatment was failing.
D. A vertical root fracture was present.
E. There were excessive occlusal loads on the tooth.

308 A 23 year old patient attends complaining of pain in an upper right molar and is keen to keep the tooth. The pain is
typically
sharp in nature, is triggered by cold and persists after removal of the cold stimulus. The tooth is not tender to
percussion; a
radiograph of the upper right first molar shows a large radiolucency extending to the pulp horn but no peri-radicular
changes.
What treatment is most likely needed in this case?
A. Oral pygiene instruction and fluoride application
B. Excavation of caries and placement of a permanent restoration
C. Root -ca na I treatment?
D. Indirect pulp cap and restoration?
E. Direct pulp cap and restoration

309 A patient attends your surgery complaining of severe pain, swelling and mobility associated with a lower first molar
tooth in
which there is a broken filling. A periapical radiograph indicates that the tooth has not been root filled and there is loss
of apical
lamina dura associated with the distal root and at the bifurcation. The periodontal bone support is good. There is no
significant
pocketing.
What is the likely diagnosis?
A. Chronic periapical periodontitis
B. An acute periodontal-endodontic periodontitis
C. A chronic periodontal-endodontic periodontitis
D. An acute periapical periodontitis
E. Pararadicular periodontitis

310 An upper incisor in a 16 year old patient has suffered trauma and the coronal tissue has been lost. The tooth
has been
endodontically treated.
How is the tooth best restored?
A. With a fibre post, direct core and crown.
B. With a direct core and crown.
C. With composite
D. With an indirect post-core from a pre-fabricated pattern and a crown
E. With a custom indirect post-core and crown

311 A 50 year old male patient has a Class III jaw relationship with an anterior open bite. It is planned to restore his
lower right
second molar, which has suffered tooth wear and fracture, with an indirect restoration. This tooth has approximately
2mm of
coronal height.
What would be the most suitable approach to restore this tooth?
A. Provide an adhesively retained gold onlay
B. Provide a conventional full crown
C. Increase the vertical dimension and provide a full crown
D. Surgically crown lengthen and provide a gold crown
E. Provide an adhesively retained ceramic onlay

312 A patient complains of a lower incisor which has been mobile for several months. The radiograph indicates a normal
level of
bony support although the periodontal space has widened. The apical bone appears normal. The tooth is tender to
pressure.
Which of the following tests and or examinations would be most likely to provide a diagnosis?
A. Masticatory muscle palpation
B. Electric pulp test
C. Occlusal examination
D. Ethyl chloride test
E. Hot gutta percha application

313 A 23 year old male presents to your surgery. He lost his upper lateral incisors some 10 years ago in a swimming pool
accident.
Since then he has been wearing a 'spoon' denture which he now feels in aesthetically unacceptable. He has sought an
opinion on
dental implants but has been told that he would need bone grafting for this to be successful and he is not prepared to
undergo this.
His dentition is excellent with no restorations and a Class I occlusion. He wants some advice on what the best treatment
might be.
Which option would you put first on your list of possibilities?
A. Two fixed - fixed resin bonded bridges using the central and canine teeth
B. Two cantilever resin bonded bridges from the central incisors and canines.
C. Two conventional fixed - fixed bridges from the canine
D. Conventional cantilever bridges from the canines
E. Cobalt chrome partial denture

314 A stabilisation splint [Michegan splint] is commonly indicated in patients needing advanced restorative dentistry and
patients
suffering from some Temporomandibular Disorders
Other than upper and lower impressions, which of the following records will be needed to construct this splint?
A. Facebow and a protrusive wax record
B. Facebow and an "Oclusal Sketch"
C. Centric Relation [Retruded Contact Position] record
D. Facebow and a Centric Occlusion [Incuspation Position] record
E.Facebow and Centric Relation [Retruded Contact Position] record _

315 A 55 year old female patient is missing her upper right second premolar and upper right first molar and also is
missing the
upper left second molar. The upper right second molar is functional and has an amalgam restoration (MOD and buccal
wall) that
requires replacing. The patient has no functional or aesthetic concerns.
What would be the treatment of choice in this situation?
A. Provide an upper removable partial denture
B. Replace the amalgam in the upper right 7 only
C. Provide a full coverage crown in the upper right 7
D. Provide a fixed bridge in the upper right quadrant
E. Provide a full coverage crown in the upper right 7 with guide planes and occlusal rests
316 Endodontic treatment has failed on an upper first molar; the patient is keen to retain the tooth. There are persistent
symptoms
from the tooth. Radiographically there is evidence of periapical radiolucency although the three canals are obturated with
good
length and compaction.
What is the best course of action?
A. Extract the tooth
B. Re-treatment with an iodine solution as irrigant
C. Re-treatment with hypochlorite solution as irrigant
D. Re-treat the tooth, looking particularly for additional canals
E. Surgical apicectomy

317 You suspect that there is occlusal caries in the lower right first permanent molar of a 10 year old child. You wish to
confirm
your suspicions.
Which diagnostic test is most commonly used in this situation?
A. Bitewing radiography
B. Electro-conductive caries monitors
C. Fibro-optic transillumination
D. Panoramic radiography
E. Visual examination of a dried tooth.

318 You notice that a 20 year old patient has marked tooth surface loss associated with the labial and palatal aspects
and incisal
edges of the upper anterior teeth. They are sensitive to hot and cold. The remainder of the dentition is mainly
unaffected
What is the likely diagnosis?
A. Attrition
B. Active erosion
C. Passive erosion
D. Abfractions
E.Abrasion

319 The best radiograph for investigating the maxillary sinus is,
A. Periapical radiograph
B. Panoramic view
C. Lateral cephaloghraph
D. Occipitomental view
E. Reverse Towne's view

320 Which is true in regard to osseo integration of implants in dentistry?


A. The osseointegration is directly between titanium and bone
B. Following insertion, implants can be immediately loaded without problem
C. The success of the implants is directly proportional to its area of contact with bone and the bond is
mechanical in nature
D. The success of the implants depends mostly on low torque preparation and insertion of the fixture
E. The success of integration is accurately investigated by immediate radiographic examination

321 In the preparation of Premolar class I cavity what is the best way of getting retention,
A. Slightly done undercut of the mesial and distal walls
B. Slightly done undercut ofthe buccal and lingual walls
C. The convergence of the cavity walls

322. In respect to Class V


A. it occurs on the buccal groove (fissure)
B. it occurs on the lingual groove (fissure)
r>; C. it is a result of bad oral hygiene

323. After the placement of a class I amalgam the patient comes back to you complaining of pain on masticating
and biting; what is
the first thing you would look at,
A. Occlusal height
B. Contacts areas

324. Child comes to your clinic with a fractured incisor 3 mm super-gingival, how would you
treat the case,
A~ Formocretasol pulpotomy
B. Calcium hydroxide pulpotomy
C. Pulpectomy
D. Direct capping
E. Indirect capping

325. What is Ante's Law about,


A. The relation between the span of the bridge and the poetics
B. The periodontal area of the abutment teeth
C. The relation between the length of the root and the abutment.

326. What is the best way to cement


Maryland bridge,
A.GIC
B. Resin
C. High compression restorative resin
D. Zinc Phosphate cement
E. Oxide Zinc and eugenol

327. The ideal length of core in the fabrication of crown and core of endodontically treated tooth is,
A. 1.5 of crown length
B. The length of the crown
C.2/3
tooth/root
length
D. 1/2, root
length

328. If aesthetic is not a concern what is the first thing to do to treat soreness under dentures,
A. Take the denture off for a week
B. Rinse the denture in nystatin
C. Apply tissue conditioner

329. While you finishing a class I cavity, the enamel is sound but you noticed in the dentine and on the Dento-
enamel junction a
brown line, what is your response,
A. You leave it and complete the final restoration
B.You extend you preparation and clean it
C.You apply a cover of varnish

330. Dental caries of the proximal surfaces are usually start at,
A. Somewhere between the ridge and the contact area
B. Just gingival to contact areas
C. Just about the gingival margin

331. What is the property of high copper amalgam


A. Reduced physical creep
B. Higher retention

332. In regards to colours what is Chroma stands for,


A. Degree of saturation of hue
B. Brightness
C. Value
D.Contrast

333. Frankfort plane extends from,


A. horizontally from Sella to nasion
B. Sagittal from
C. Horizontally from point on superior aspect of external auditory meatus to
orbitale

334. Which of the following local anaesthetic is indicated in case of the need to long acting one after a
surgical operation,
/~ A. Lidocaine
B. Mepivacaine
C. Bupivacaine (Marcaine)

335. In respect to Lidocaine 2% with 1:100000 vasoconstrictor,


A. The toxic threshold is 22ml
B. 8.8 ml is the maximum you can give in one session

336. Which one of this restorative method will be LEAST compromised by a


core,
A. Amalgam
B. Composite
C. GIC
D. Cast gold

337. In preparing a very small proximal amalgam cavity on a molar tooth what
would consider,
A. Extend the cavity to the gingival margin
B. Extend the cavity beyond the contacts areas
C. Achieve at least 2mm in dentine
D. Extend cavity just beyond dento enamel junction

338. What is true about partial dentures,


A. They cause an immediate changes in the oral plaque behaviour
B. Night wearing of dentures reduces plaque accumulation
C. Relieving the gingival area reduces the gingival enlargement.

339. The biting load of denture base to tissues compared to teeth are,
A. Ten times more
B. Ten times less
C. Equal

340. Compound is,


A. Very accurate compression material
B. Thermoplastic material
341. The different between normal stone and the dye stone is,
A. In the particles size
B. The amount of water

342. A patient comes to you with medium pain of tooth filled with Composite resin as a result of cold or hot drinks,
what your initial
management will be,

A. Remove the restorative material and start an Endontic treatment


B. Remove the restorative material and place a sedative temporary material
C. Place a coat of bonding material on the old composite

343. Throbbing pain increases with heat and cold stimuli, the MOST probable
diagnosis is,
A. Cyst
B. Occlusal trauma
C. Advanced pulpitis

344. I n making your custom trays which of the following is true,


A. A uniform thickness is required
B. Perforation is better
C. Only adhesive is better than perforation

345. The most common cause of caries in children is,


A. Soft diet
B. High intake of carbohydrate
C. Poor oral hygiene

346. The best storage media for avulsed tooth is,


A. Saline
B. Milk
C. Water
D. Saliva

347. An occlusal approaching clasp TIP,


A. Should occupy a predetermined undercut
B. Contact the tooth under the survey line
C. Rigid

348. In the construction of partial denture the surveyor is not used to,
A. Contour the wax as part of the fabrication of the working cast
B. Locate the guide planes
C. Determine the location of indirect retainers
D. Identify any undesirable undercuts

349. The main purpose of finishing the enamel walls is,


A. Remove loose enamel rods
B. Provide a better surface for the adoption of restorative material

350. A female patient comes to you complaining of persistent pain in heavily restored central incisor; you suspect pulpitis
and you
have been told that she is in transit leaving by plane next day. Your treatment will be,
A. Remove filling and place a sedative dressings
B. Pulp extirpation and obturate with Ledermix dressings
C. Prescribe analgesics and systemic antibiotic

351.which of the following is true regarding TMJ dysfunction,


A. It is always due to arthritis, should be treated with NSAIDS before attempting
surgery
B. Raising bite increases the space in the joint and should be attempted before
surgery
C. It is mostly due to the medial movement of the condylar head over the glenoid fossa

352.The location of Class V is in,


A. The bucca I pit /fissu re
B. The occlusal surface
C. The cervical third

353. Occlusal cavity with extension of the buccal fissure is classified as,
A. Class II
B. Class III
C. Class I

354. Which of the following does not affect the elasticity of retentive clasp?
A. Length of the arm
B. The cross section shape

C. The material used


D. The undercut area

355. Following calcium hydroxide pulpotomy, the dentist would expect dentine bridge to form at,
A. The exact level of amputation
B. Level some where below the amputation
C. Half way between amputation and apex
D. At the apical region of the tooth

356. In the construction of a full veneer gold crown, future recession of gingival tissue can be prevented or at least
minimised by,
A. Extension ofthe crown 1 mm under the gingival crevice
B. Reproduction of normal tooth incline in the gingival one third of the crown
C. Slight over contouring of the tooth in the gingival one fifth of the crown
D. Slight under contouring of the tooth in the gingival one fifth of the crown

357. A partial denture that seats on the master cast but fails to seat correctly in the mouth is a result of,
A. Contraction of the metal framework during casting
B. Insufficient expansion of the investment material
C. Distortion of impression
D. Failure to block out unwanted undercuts

358. Which of the following muscles may affect the borders of mandibular complete denture?
A. Mentalis
B. Lateral pterygoid

r>. C. Orbicularis oris


D. Levator oris
E. Temporalis

359. What is correct in regards to high copper amalgam,


A. Reacts and strengthens the amalgam by its dispersion properties
B. Reacts to form copper-tin phase thereby eliminating the tin-mercury phase
C. Reacts to form copper-silver phase thereby eliminating the silver mercury phase
D. Reacts and strengthens the amalgam by its grain diffusion

360. What is the main purpose of using corticosteroids in pulpal obturation material?
A. For their antibiotic action
B. For their antiinflammatory action
C. To relief pulp pressure

361. Which of the following statements is incorrect regarding Smoker's Keratosis?


A. Typically affects the hard palate
B. Minor mucous glands are swollen with red orifices
C. There is a little regression if smoking is stopped

362. 13 years old boy comes to you with excessive hyperplasia of the gingiva as a result of Phenytoin what is your
management,
A. Stop the medication
B. Force a strict oral hygiene and surgical removal of excess gingival tissues
C. Debridement and conservative approach

363. White man 56 years old comes to you with a brown spot on his gingiva and another one on his oral mucosa,
when taking the
history he mentioned a weight and memory lost. He as well complains of headaches. What is your most probable
diagnosis,
A. Addison's disease
B. Hyperthyroidism

364. While removing the second primary molar of 9 years old child, the apical Y. of the root fracture and stay in the
socket,
A. You will just leave it and observe it
B. You take surgically by a lingual flap
C. You try to take out by using a root apex elevator
D. You use a fine end forceps to take it out

365 What is the most important factor to reduces dental irradiation,


A. Speed of film
B. Collimation
C. Filtration
D. Cone shape and length

366. With view to Nitrous Oxide what is the major pharmacological problem?
A. Contraindicated in pregnancy
B. Contra indicated in cardiac dysrhythmias

367. Which of the following is an expansile lesion of the oral mucosa,


A. Keratocyte
B. Radicular cyst
C. Cementoma

368. The concentration of Fluoride in the


topical NaF
A.2%
B.5%
C. 8%
D.10%

369. patient in your dental chair shows chest pain, weak pulse and dysponea, what is your initial
management,
A. Give a nitro-glycerine tablet and keep the patient up seated'
B. Put the patient in supine position
C. Do nothing and wait until the symptoms go

370. What are two teeth connected at the


cementum called,
A. Concrescence
B.
Dilace
ration
C.
Gemin
ation
D.
Fusion

371. Developer contaminated with other chemical and was not mixed pro perlv. What is the effect on
the X-ray film?
A.Too dark film -
B. Light film
C. Foggy

372. Which part of the cranium is considered as the most stable area,
A. Frankfort plane
B. Occlusal plane
C. Anterior cranial plane
D. Anterior nasal to gnathion.

373.The difficulty of placing matrices on deciduous dentition is a result of,


A. The small mouth of kids which result in problem keeping the matrices in their mouths
B. The occlusal convergence of the deciduous teeth

374. Branchial Cleft cyst is located,


A. Medial to the neck
B. On anterior border of the Sternocleidomastoid muscle
C. Shows when swallowing

375. the most common way of oral carcinomas to other tissues is/are
A. Lymphatic
B. Invasion and blood
C. Lymphatic and invasion
D. Blood and lymphatic

376. The main purpose of periodontal treatment is,


A. Elimination of plaque and calculus
B. Elimination of periodontal pockets
C. Reformation of all the periodontal ligaments
D. The elimination of all occlusal trauma
377. What is the approximate unstimulated salivary flow rate,
A. 2 ml/min
B.0.2 ml/min
c. 0.02 ml/min D. 20 ml/min

378. The thermal and electric pulp tests will,


A. Give an accurate indications of the pulp status
B. The patient's response will be either pain or no pain
C. The patient can differentiate between cold or hot stimuli

379. Immediately after the extraction of lower molar the patient complains of post operation bleeding and pain, how
would
manage this,
A. Prescribe analgesics and ask the patient to follow a strict oral hygiene
B. Administer 5% Marcaine Local Anastatic, prescribe analgesics and pack the socket with alvogyl
C. Administer 5% Marcaine Local Anastatic, suture the socket and prescribe analgesics
D. Suture and give pressure packs

380. Gracey curette is characterized by,


A. The blade and the shank form 90Q angle
B. Can be used on both sides
C. Can be used on any tooth surface
D. It is specific for each surface of the tooth

381. The removable partial denture requires relining what is would be the most appropriate action,
A. take a new impression by asking the patient to occlude on it
B. Provide equal space between denture and gingival tissues.
C. Make sure the framework and retainers are seated in place before taking impression

382. In regards to dentine strength, which is the right sequence,


A. Affected dentine> Sound dentine> Infected dentine
B. Sound dentine> Affected dentine> Infected dentine

383. Symptoms free patient comes to you after four weeks of an endodontic treatment and you find on radiograph the canal
is over
filled with what it seems to be a cone of Gutta Percha lmm beyond the apex with a radiolucent small area. What is your initial
management?,
A. Start apiectomy through a flap and surgery
B. Obturate the root canal
C. Ask for a recall and observe in three months time
D. Seal the pulp chamber and keep it under observation

384. After obturation and on X-ray you notice the obturation materials are lmm beyond apex. What is your first
management?
A. Refill the canal
B. Pull the GP cone about lmm out and take a new X-ray
C. Leave it as it

385. 2.21mg NaF


contains,
A. lmg fluoride
B.2 mg
C. 0.5 mg

386. Stiffness of material are measured by,


A. Proportional unit
B. Modules of elasticity
C. Stress/ strain

387. Four years kid shows at your clinic with open bite as a result of thumb sucking, you notice a delayed speech ability
what would
be your first management,
A. Refer to a speech therapist
B. Apply a removable habit inhibitor denture
C. Apply a removable habit inhibitor denture and educate the parents about it so the kid will not be taking it off so often

388. Two central incisors on a radiograph are showing with what looks like eye drop radiolucency. You decided to start
endodontic
treatment on these teeth but you tried to open access to the root canal you find clearly closed orifices with what look like
secondary dentine. What is your initial management?
A. Leave as it and start a permanent restoration.
B. Start systemic antibiotic
C. Try to ream and file canals
389. A patient with no positive history came along for scaling. The moment you pick up the scaler you punch your
finger, what
should you do?
A. Complete the procedure as nothing has happened
B. Check patient's blood for Hepatitis B antibody HBsAb
C. Check patient's blood for Hepatitis B antigen HBsAg
D. Check dentist's blood for Hepatitis B antibody HBsAb and HIV antigen HIVAg
E. Check dentist's blood for Hepatitis B antigen HBsAg and HIV antibody HIVAb

390 when probing for periodontal disease the tip of the probe will be,
A. At the coronal end of junctional epithelium
B. At the top of the gingival calculus

391. After the initial development stage and in the absence of pathology, a the size of the pulp chamber has been
reduced by,
A. Deposition of primary dentine
B. Deposition of secondary dentine
C. Reparative dentine
D. Pulp fibrosis

392. The most desirable outcome of endodontic treatment is,


A. The healing of the alveolar bone
B. The deposition of cementum at the apex
C. Formation of fibrous capsule around the apex

393. What is NOT related to the normal aging process?


A. Progressive bone loss
B. Reduced elasticity of muscles
C. Decreased elasticity of the skin
D. Lower pain threshold

394. The most accurate finding of pulpal pathology


A. Radiolucency on the apical region
B. Pain on hot or cold drinks
C. The absence of response to pulp testing

395 For dental caries to progress in dentine, the dentine must contain soluble collagen
A. enamel must contain glycoproteins
B. diet must contain simple carbohydrate
C. diet must contain polysaccharides
D. pulp must contain complement

396 A patient on the dental chair has cardiac arrest which is INCORRECT,
A. Observing the vital signs and check that the air way is clear is at high importance
B. Expired air has 15% 02 only, and cardiac compressions achieve 30-40% of cardiac output
C. Intermittent positive pressure at the rate of 40/min will reduce the chances of cerebral hypoxia
D. Intermittent positive pressure is better than mouth to mouth when it has been given at the same rate.

397. A 9 year old boy has a small white discolouration on his maxillary central incisor. The lesion is most probably,
A. Hypocalcification due to trauma of the primary predecessor
B. Hypoplasia due to acute systemic infection when 6-12 months old
C. Defect during the histo differentiation stage of development
D. Defect during the morho differentiation stage of development

398. The best method to take X-ray of the maxillary sinus is,
A. Periapical radiograph
B. Panoramic view
C. Lateral cephaloghraph
D.Occipitomental view
E. Reverse Towne's
view

399. Which of the following is not a part of the fully formed enamel organ,
A. Outer enamel epithelium
B.Inner enamel epithelium
400.18 years old female her weight is 52Kg and she is 163cm tall. On dental examination erosion of teeth on the most of her
lingual surfaces is clearly showing. Dietary history revealed a daily rate of 5000 to 7000 Kcal/day. What is most probable
would be
her case?
A. Alcoholism
B. Drug abuse
C. Bulimia
D. Excessive smoking
E. Diabetic mellitus type I

401. Which one of the following is true in regards t osseointegration implants in dentistry?
A. Fibrous tissues are formed and integrated directly between titanium and bone
B. Following insertion, implants can be immediately loaded without problem
C. The success of the implants is directly proportional to its area of contact with bone
D. The success of the implants depends mostly on low torque preparation and insertion of the fixture

402. Generalised lost of tooth structure by chemical means called,


A. Erosion
B. Attrition

403. On X-ray, the buccal roots of 16 is considerably elongated; this is a result of,
A. Too great vertical angulation
B. Inadequate vertical angulation
C. Excessive object film distance

404. The principle muscle responsible for the opening of the mouth is,
F. Mylohyoid
G. Anterior temporal
H. Posterior temporal
I. Anterior belly of digastric

405. Denture stomatitis is commonly associated with,


A. The continuous wearing of removable orthodontic appliances in otherwise healthy patient
B. The proliferation of hypertrophic tissue at the denture periphery
C. The overgrowth of some constituents of oral normal microflora
D. Allergy to denture base material

406. Which of the following is NOT characteristic of Down's syndrome?


A. Decreased neutrophil function
B. Macroglossia
C. Macrodontia
D. An increased susceptibility to periodontal disease
E. Congenitally missing teeth

407. Which of the following is the best evidence that a previous periodontal treatment is successful?
A. The patient keeps a 3 month recall appointment
B. There is no extrinsic stain
C. The patient demonstrates good understanding of brushing and flossing techniques
D. There is no bleeding on probing

408 A 65 year old female presents at your surgery complaining of an extremely sharp pain of a few seconds duration which
arises
whenever she touches an area of skin above the right hand side of her upper lip adjacent to the angle of her mouth. The
patient is
fit and well and is not taking any medication. You make a diagnosis of trigeminal neuralgia.
What is the drug of choice for treating trigeminal neuralgia in such a patient?
A. Carbamazepine
B. Oxcarbamazepine
C. Gabapentin
D. Phenytoin
E. Ibuprofen

409 A patient on examination was found to have swollen gingiva around a crown that had been present for several
years. The
papillae were particularly enlarged.
What is the most important feature of a crown that may be responsible for this?
A. Material of the Crown
B. The occlusion

C. proximal Contour
D. Labial Contour
E. Surface finish

410 A 13 year old girl presents with an unerupted permanent canine and a retained primary canine. You cannot
palpate the
unerupted canine in the buccal sulcus and you are uncertain as to whether it is displaced palatally or in the line of the
arch.
Which single radiographic view would be most helpful in locating the unerupted tooth?
A. Bitewing
B.DPT
C. Lateral oblique
D. Single periapical
E. Upper anterior occlusal

411 A 30 year old man with unknown allergy to latex goes into anaphylactic shock whilst being treated in the dental
surgery.
Which drug and route of administration is of most benefit in this situation?
A. Hydrocortisone - orally
B. Chlorphenamine - intramuscularly
C. Chlorphenamine - orally
D. Epinephrine - intravenously
E. Epinephrine - intramuscularly

412 A 62 year old female presents at your surgery complaining of a persistent, dull ache affecting her upper left 4. The
pain is
present all the time but varies in its severity although the patient cannot think of any exacerbating factors. It is not
relieved by
analgesics. Over the past 18 months the patient has had several teeth extracted from the upper left quadrant. Each
extraction
brings about temporary relief of her symptoms only for them to recur in an adjacent tooth.
What is the most likely cause of the patient's pain?
A. Acute/reversible pulpitis
B. Dentine sensitivity
C. Chronic/irreversible pulpitis
D.Atypical
odontalgia
E. Trigeminal
neuralgia

413 A 25 year old patient attends your surgery complaining of a swelling at the angle of the mandible. A radiograph shows
a uni-
locular radiolucency associated with the crown of an unerupted wisdom tooth.
Which of the following is the most likely diagnosis?
A. Radicular cyst
B. Dentigerous cyst
C. Lateral periodontal cyst
D. Ameloblastoma
E. Odontogenic keratocyst

414 A 70 year old female, who suffers with persistent looseness of her lower complete denture, is considered for implants.
She will
require radiological evaluation of the potential implant sites.
Which would be the most appropriate radiological investigation at this stage?
A. MRI scan of the mandible
B. CT scan of the mandible
C. Periapical radiographs of the mandibular anterior region
D. True lower anterior occlusal view
E. Panoramic radiograph

415 Six months ago you saw a child patient, then aged 9 years. His upper right maxillary canine was palpable in the labial
sulcus but
the upper left was not. The situation is now unchanged, so you have taken two periapicals of the non-palpable tooth. They
both
show that there is some resorption of the CI root but the permanent canine appears somewhat mesially angled and is more
mesial
on the more mesially positioned film.
What is your the best course of action?
A. Keep a careful watch on it and take another x-ray in 6 months.
B. Refer to an oral surgeon for early exposure of the permanent canine.
C. Refer to an oral surgeon for early removal of the permanent canine before it damages the lateral incisor.
D. Refer to an orthodontist for a treatment plan.
E. Wait and watch' until the child is 11.

416 An apical radiolucency (2mm) is noticed as an incidental radiographic finding associated with the apex of the
mesiobuccal root
of the lower right first molar. The tooth has been root filled but is 2mm short of the radiographic apex. There are no other
clinical
or radiographic findings and the patient is fit and well. What is the most appropriate course of action?

A.Extract the tooth.


b.Redo the root filling
c.Perform periapical surgery.
d.Advise the patient of the situation and monitor clinically and radiographically
e.Prescribe antibiotics then review

417 A 43year old patient is missing on the upper right the first premolar and molar. He has good oral hygiene and requests a
fixed
replacement for these teeth. The other teeth on the same side are all moderately restored with MOD amalgam restorations
and
are vital, except the canine, which has a very large restoration and is root-filled. He has group function. Radiographs show
a large
sinus cavity and no peri-apical pathology. What would be the restoration of choice for replacement of the missing teeth?
A. Implant supported crowns
B. A conventional fixed bridge using the 7 and 5 as abutments
C. Two conventional cantilevered bridges, using the 7 and 3 as abutments
D. A resin-bonded bridge, using the 7 and 5 as abutments
E. A conventional fixed-moveable bridge using the 7 and 5 as abutments

418 Epidemiological studies have shown that dental decay is normally greater amongst northerners and those in socially
deprived
circumstances. Which of the following would be most effective in reducing cariesJn a high risk population?
A. Brushing with fluoride toothpaste
B. Publicity campaign
C. Fluoridation of the water supply
D. Dietary advice via schools
E. Fissure sealant provision

419 A 58 year old male presents at your surgery complaining of a sharp pain of no more than 30 minutes duration arising
from his
upper left molar region. The pain is brought on by cold stimuli but persists after-the stimulus is removed. It does not seem to
occur
spontaneously. He has tried taking paracetemol and this does temporarily stop the pain from recurring. The upper left 6'
reacted
to a lower current on electronic pulp testing than the upper right 6, upper left ~ or the lower left molars.
What is the most likely cause of the patient's pain? -
A. Acute/reversible pulpitis
B. Dentine sensitivity
C. Chronic/irreversible pulpitis
D. Periapical periodontitis
E. Trigeminal neuralgia

420 A 60 year old patient attends your surgery complaining of a sore mouth. He has Type II diabetes well controlled by
diet and
metformin. On examination white patches which cannot be removed are present on his buccal mucosa:
What is the most likely diagnosis?
A. Frictional keratosis
B. Leukoplakia
C. Lichen Planus
D. White sponge naevus
E. Candidosis

421 A 60 year old female attends your surgery complaining of soreness affecting her gingivae. No other area of her oral
mucosa is
affected but she has noticed an itchy rash on the flexor surface of her forearms. She is fit and well and is not taking any
medication. Scattered purple/red papules each about 4mm in greatest dimension are present on the flexor surface of her
forearms
and on intraoral examination a desquamative gingivitis is present. Based on the above findings what is your diagnosis?
A. Mucous membrane pemphigoid
B. Lichen planus
C. Pemphigus vulgaris
D. Erythema multiforme
E. Lichenoid drug reaction

422 An 80 year old patient presents with an ulcer in the floor of the mouth. This has been present for several months and
has not
responded to conventional treatment. An incisional biopsy is taken. Which of the following histological changes in the
epithelium
confirm a diagnosis of squamous cell carcinoma?
A. Hyperkeratosis
B. Acanthosis
C. Dysplasia
D. Invasion
E. Discontinuous epithelium

423 A 35 year old patient complains of swollen gums. This has been present for several years. What is the commonest
cause of this complaint?
A. Cyclosporin therapy
B. Vitamin C deficiency
C. Chronic Gingivitis
D.Atenolol therapy
E. Pregnancy

424 A 45 year old patient attends the dental clinic complaining of a clicking jaw. Examination reveals a reproducible click
of the
right TMJ when opening wide. Upon asking the patient to open wide, close with incisors edge-to-edge and then open and
close to
this position, the click is absent. From the options below, which one is the most likely diagnosis?
A. Myofascial pain
B. Disc displacement with reduction
C. Disc displacement without reduction
D. TMJ osteoarthritis
E. Arthralgia

425 Cigarette smoking is considered to be the most important factor next to microbial plaque in periodontal disease
progression.
Which of the following is the most important factor in the disease progression in smokers?
A.Smokers have drier mouths than non-smokers .,
B. Smokers have poorer oral hygiene than non-smokers.
C. Nicotine will impair the chemotactic and phagocytic properties of PMNs.
D. The gingival blood flow is reduced in smokers.
E. Smokers alter the oral environment encouraging the growth of anaerobic bacteria

426 An adult patient attends your practice complaining of pain and swelling associated with a previously restored
upper first
premolar tooth. The pain has been present for a number of days and is no longer responding to analgesics. His
dentition is
otherwise well maintained and his periodontal health is good.
What is the most appropriate approach to treatment?
A. Antibiotics and analgesics.
B. Extract the tooth
C. Carry out a pulpotomy. Temporary dressing.
D.Carry out a pulpectomy. Temporary
dressing
E. Establish open drainage

427 A patient reports that his post crown has fallen out. This crown had been present for many years. You note that there
appears
to be a hairline vertical fracture of the root. The tooth is symptomless.
What is the most sensible approach to treatment?
A. Replace the post crown using a resin-reinforced glass ionomer material
B. Replace the post crown using a polycarboxylate cement
C. Replace the post crown using a dentine bonding agent and a resin- reinforced glass ionomer material
D. Replace the post crown using a resin composite luting agent
E. Arra nge to extract the tooth

428 A patient says that he does not like the appearance of his previously root filled upper central incisor tooth. His
dentition is
otherwise well maintained and his periodontal health is good. The tooth appears to be darker than the adjacent teeth.
What is the most appropriate approach to treatment?
A. Provision of a post crown
B. Provision of an all ceramic crown
C. Provision of a metal bonded to ceramic crown
D. Carry out a non vital bleaching procedure
E. Provision of a porcelain veneer

429 A 20 year old patient attends your surgery for the first time. You suspect that he may have proximal caries as he has a
frequent
sugar intake. Which of the following is the most accurate method of diagnosing proximal caries in a lower molar tooth in this
young
adult?
A. Clinical history
B. Periapical radiograph
C. Bitewing radiograph X
D. Digital image
E. Electronic resistance measurements

430 A 25 year old male attends for the first time complaining of sensitivity of a number of teeth. On examination, the
occlusal
surfaces of all the teeth are worn with obvious wear facets on the canines and premolars. Posterior amalgam
restorations are
proud of the surrounding tooth.
What would be the first stage management?
A. Take impressions for study models
B. Prescribe fluoride mouth rinse
C. Replace the amalgam restorations
D. Dietary analysis
E. Placement of resin sealant to sensitive teeth

431 A 35 year old male patient who admits to grinding his teeth at night has a number of wedge- shaped cervical (Class V)
lesions
on his upper premolar teeth. These are causing some sensitivity and are approximately 3mm deep. What is the correct
management option?
A. Provide tooth brushing instruction and fluoride
B. Restore the lesions with compomer
C. Restore the lesions with micro-filled composite
D. Restore the lesions with a hybrid composite
E. Restore the lesions with conventional glass-ionomer

432 A patient attends with pain of four days duration in a carious upper molar tooth. The pain is constant and is not
relieved by
paracetemol. Sleep has been disturbed by the pain. The tooth is tender to percussion and gives a positive response to
Ethyl
Chloride. What is the most likely diagnosis? .,
A. Pericoronitis
B. Apical periodontitis
C. Marginal periodontitis
D. Reversible pulpitis
E. Irreversible pulpitis

433 You are trying in a partial chrome denture framework which fails to seat properlv. It fits the
master cast.
What is the most likely cause of this problem?
A. Insufficient expansion of the investment material
B. Distortion of the impression
C. Contraction of the metal framework during casting
D. Failure to block out unwanted undercuts
E. Complex denture design

434 Bitewing radiography is the main special text used to help in diagnosis of proximal caries. The performance
(accuracy) of a
diagnostic test like bitewing radiography can be expressed in terms of sensitivity and specificity.
Which of the following is a reasonable summary of the diagnostic accuracy of bitewing radiography for proximal caries
diagnosis?
A. Moderate sensitivity and low specificity
B. Moderate sensitivity and moderate specificity
C.Moderate sensitivity and high
specificity
D. High sensitivity and moderate
specificity
E. High sensitivity and high specificity

435 You are interested in finding out what the risk indicators are for a rare form of oral cancer and decide to undertake a
study to
examine this.
What type of study would be the most appropriate for addressing this issue?
A. Cohort
B. Prevalence study
C. Clinical trial
D. Case-control study

436 You take a panoramic radiograph of a patient and discover a well-defined, corticated radiolucent area below the
inferior
dental canal just anterior to the mandibular angle. What is the most likely diagnosis?
A. Radicular cyst
B. Stafne bone cavity
C. Metastatic carcinoma of the breast
D. Adenomatoid odontogenic tumour
E. Complex odontome

437 You take a panoramic radiograph out of a patient's records but you find that the film has a low density and poor
contrast.
Which of the following errors could lead to low density and poor contrast?
A. Too long a development time
B. Developer temperature too high
C. Developer temperature too low
D. Inadequate fixation E.
438 You want to evaluate the effectiveness of using tetracycline as an adjunct to scaling and root planing for the
treatment of
chronic periodontitis. What type of primary study design would be most appropriate for addressing this topic?
A. Cohort study
B. Non-randomised controlled trial
C. Randomised controlled trial
D. Case-control study E. Case-series

439 Radiation protection of patients is partly dependent upon equipment factors (x- ray set and film or digital
system). The
different factors that can be changed vary in financial cost to the dentist and in their effectiveness in cutting x-ray
dose.
Which of the following provides the most cost-effective means of minimizing patient radiation dose in dental intraoral
radiography?
A. Constant potential ('DC) x-ray set
B. Rare earth filtration
C. Lead apron
D. D speed film
E. F speed film

440 A 42 year old man presented with a firm fixed swelling in the right preauricular region which had been enlarging over
the last
month. He had also developed a right sided facial palsy.
What is the most likely diagnosis?
A. Adenoid cystic carcinoma
B. Squamous carcinoma
C. Sebaceous cyst
D. Branchial cyst
E. Pleomophic adenoma

441 A 14 year old patient attends with a decayed and extensive hypoplastic LL7: He is a very irregular attender with
poor oral
health habits. A radiograph shows the presence of an unerupted LL8 and the LL6 is sound.
What would be the most appropriate long-term treatment for this tooth?
A. Amalgam restoration
B. Antibiotics
C. Extraction
D. Root canal therapy
E. Sedative dressing

442 A 7 year old boy has previously had all primary molars restored and a pulpotomy on upper right E. He has an early
mixed
dentition with lower lateral incisors erupting. There is a midline diastema of 2 mm. The upper right E has become
symptomatic and
requires extraction.The most likely long term effect of the extraction on the occlusion is:
A. Early eruption of the second premolar.
B. Loss of upper central line.
C. No significant effect.
D. Overeruption of the lower right teeth.
E. Potential crowding in the upper right quadrant

443 Which of the following is the ideal treatment for a degree II furcation involvement of a mandibular molar?
A. Tunnel preparation
B. Root resection
C. Furcation pia sty
D. Extraction
E. Guided Tissue Regeneration

444 You examine a patlent and find BPE code 4 in all sextants. Radiographs show generalised horizontal bone loss with a
minimum
of 50% of bone support remaining on all teeth.
Which of the following is the most important factor when considering the prognosis for the teeth?
A. Age of the patient
B. The Oral Hygiene Status
C. Bleeding on Probing Score
D. Mobility E. Gingival Recession

445 A 30 yr-old patient attends complaining of occasional pain from the lower left quadrant. Clinical examination reveals
an
extensively restored dentition with generally good oral hygiene. There is no significant periodontal pocketing other than
an
isolated defect in the region of the furcation of lower left first molar which is non-mobile. The gingival tissue in this area
appears
erythematous and slightly hyperplastic with a purulent exudate on probing.
From the list below, which is the most appropriate 'next step?
A. Obtain a radiograph

B. Biopsy the gingival tissue


C. Remove the restoration
D. Vitality testing
E. Prescribe antibiotics
446 A 40 yr old patient had root-canal treatment to his upper first molar. This was performed 6 months ago using contemporary
techniques under rubber dam and was crowned after completion of treatment. He attends complaining of continued discomfort
from this tooth. Radiographic examination shows each of the three roots to be obturated with a well-condensed filling to the full
working length though there is no evidence of in-fill of the periapical lesion when compared to the pre-op view. There is crestal
bone loss and no furcal involvement. What is the most likely cause of the continued problem?
A. Extra-radicular infection
B. Contamination of canal(s) with E.faecalis
C. Uninstrumented canal
D. Vertical root fracture E. Perio-endo problem

447 A mother is concerned that her child's adult upper front teeth have not erupted and asks your advice.
What is the usual age in years for the upper permanent central incisors to erupt?
A. 5 years.
B. 6 years.
C. 7 years.
D. 8 years.
E. 9 years.

448 You decide to refer an eight year old child to the oral surgery department in your local hospital for extractions under a general
anaesthetic. What key reason for asking for a general anaesthetic would you Rut in the referral letter?
A. Parents request GA.
B. Failed to complete treatment under inhalation sedation.
C. Child would not accept local analgesia
D. Parents think hospital more convenient.
E. Not one of my regular patients.

449 An eight-year-old boy presents with pain of three days duration that has kept him awake. On examination you see a grossly
carious lower left 6 and some associated buccal swelling.
Which of the following is the most appropriate to give immediate relief of his pain?
A. Extract the LL6.
B. Gently excavate the.cartes and obtain drainage.
C. Give antibiotics.
D. Incise any swelling
E. Refer for-general anaesthetic

450 A 10 year old girl tripped and-knocked out her upper incisor 10 minutes ago. She is holding it in her hand.
What is your most effective immediate treatment?
A. Accept the tooth is lost and do nothing.
B. Refer to specialist.
C. Re-plant immediately.
D. Root fill, clean and re-plant. ~
E. Sterilise root in alcohol and re-implant. .
'
451 You are treating a patient who has a long history of recurrent episodes of oral candidosis. His mouth has
recently become sore
once again and you want to prescribe some antifungals for him. You check his medical history and find that he suffers from atrial
fibrillation and is taking warfarin.
Which of the following drugs would be most appropriate to treat this patient's candidosis?
A. Metronidazole
B. Nystatin
C. Fluconazole
D. Miconazole
E. Amoxycillin

452 A dental company has claimed that ozone is better than conventional methods for treating decay in peoples' mouths.
Which of the following would provide the best evidence to support these claims?
A. Systematic review of Randomised Controlled Trials
B. Randomised controlled trial
C. Cohort study
D. Case/control study

453 A GDP in Manchester randomly selects 5000 patients, aged between 45 and 55 years, who are free from
periodontitis. He
determines that 300 of them are smokers. He follows all 5000 patients for 10 years, by which time 150 had developed
periodontitis. Of the patients with periodontitis, 60 had previously been identified as smokers and 90 as non-smokers.
What type of study is this?
A. Case-se ries
B. Prevalence study
C. Clinical trial
D. Case-control study
E. Cohort

454 Randomised controlled trials enable us to establish cause and


effect.
The main reason for using this study design is to:
A. Ensure that making a Type II error is minimised.
B. Ensure that test and control groups are similar.
C. Ensure that test and control groups are of equal size.
D. Help recruitment of patients into the trial.
E. Provide sufficient power for the study.

455 A 53 year old male patient presents with an asymptomatic white patch on the ventral surface of his tongue. He has
smoked 35
cigarettes a day since he was 17. An incisional biopsy ofthe white patch is likely to show which histological feature:
A. Basal cell liquefaction
B. Hyperkeratosis
C. Saw tooth rete ridges
D. Acantholysis
r>. E. Basal cell hypoplasia

456 A 46 year old female presents with a slowly enlarging painless firm swelling in the hard palate to the left of the
midline.
The most likely diagnosis is:
A. A dental abscess
B. Torus palatinus
C. Osteoma
D. Pleomorphic adenoma
E. Canalicular adenoma

457 A 29 year old man has a prosthetic (mechanical) aortic valve. He had bacterial endocarditis five years ago. He now
requires
removal of his upper and lower right third molars. Which antibiotic prophylaxis is recommended prior to the procedure?
A. Amoxicillin and clindamycin
B. Metronidazole and cephalexin
C. Amoxicillin and gentamycin
D. Amoxycillin and vancomycin
E. Erythromycin and vancomycin

459 A 24 year old man presents with a loculated cystic radiolucency in the lower third molar area approximately 3cm in
diameter.
There is no tooth associated with the cyst and there is no bucco-lingual expansion of the mandible. There is no resorption
of the
roots of the overlying second molar tooth.
How would this be treated?
A. Marsupialisation
B. Enucleation and ethyl chloride
C. Block resection
D. Enucleation and Carnoy's solution
E. Enucleation and formalin solution

460 A 21 year old female presents for the first time to your practice. She is very upset with the appearance of her upper
left
central incisor. On examination you find healthy oral hard and soft tissues and excellent oral hygiene. On close
examination you
can see that the upper left central incisor is slightly greyer than the upper right central incisor and has a composite
restoration
placed palatally. What is the most appropriate form oftreatment given the information you have?
A. Bleaching with carbamide peroxide in custom formed trays of upper and lower arches
B. A bonded crown
C. A composite veneer
D. A porcelain veneer
E. Non-vital bleaching with carbamide peroxide

461 EDTA (ethylene diamine tetra-acetate) has useful roles in certain situations in clinical
dentistry.
When would you use EDTA?

A.As a root end filling material


B. As a pulp capping agent
e. As a root canal chelating agent
D. As a mouthwash
E. As a dentine bonding agent

462 A patient presents with a history of clicking from their temporomandibular joint. This click occurs mid way through the opening
cycle and is consistent. There is some pre-auricular pain and the lateral pterygoid muscle on the affected side is tender to resisted
movement test. There is no trismus and the click is not present when the patient opens from an incisor edge to edge relationship,
instead of her normal Class I occlusion. The patient would like treatment.
The most appropriate occlusal splint for this patient would be:
A. Stabilisation splint
B.Localised Occlusal Interference Splint
e. Bite Raiser
D. Soft Bite Guard
E. Anterior Repositioner Splint

463 Glass ionomer cements are used in restoring Class V cavities.


Which of the following constituents are most likely to be present in glass ionomer'cements?
A Alumino-silicate glass and phosphoric acid
B. Phosphoric acid and zinc oxide
e. Polyacrylic acid and zinc oxide
D. Aluminosilicate glass and polyacrylic acid
E. Itaconic acid and zinc oxide

465 A patient presents with a history of a post-crown having fallen out. The post- crown was originally placed fifteen years ago
and had been successful up until four months ago since when it has come out and been recemented four times. At recementation
there was no evidence of any caries. The patient had been a regular attender and not needed any restorative treatment for the last
eight years. Which of the following is the most likely cause for the failure of th"fs crown?
A. The post was to narrow
B. The post was to short.
e. The root canal treatment was failing.
D. A vertical root fracture was present.
E. There were excessive occlusal loads on the tooth.

466 In dental epidemiology, indices are used to measure the oral health of a population. The DMF index is commonly used to
measure the prevalence and severity of dental caries.
What is the main limitation of this approach?
A. It does not allow statistical analysis.
B.It gives equal weight to decayed, missing and filled teeth.
e. It is difficult to calibrate examiners.
C. It is reversible.
D. There is no gold standard

467 The biological process by which the architecture and function of the lost tissue is completely restored".
In Periodontology this is a definition of which of the following
A. New Attachment
B.Regeneration
e. Repair
C. Primary Healing
D. Reattachment

468 There has been much debate regarding the effectiveness of fluoride in water for preventing tooth decay. A systematic review
of the evidence drew conclusions as to the reductions in decay which can be expected.
Fluoridation of public water supplies:
A. Reduces tooth decay by 10%.
B.Reduces tooth decay by 20%.
e. Reduces tooth decay by 30%.
C. Reduces tooth decay by 40%.
D. Reduces tooth decay by 50%.

469 An obese 40 year old who takes metformin is seen at the end of a morning clinic. After administration of local analgesia in the
supine position the patient complains of feeling unwell. She is pale and sweating and is confused. No pulse was detectable. She
was given oxygen and maintained in the supine with no improvement in condition.

What would be the most appropriate drug to administer next?


A. Glucagon
B. Glucose
C. Adrenaline
D. Hydrocortisone
E. Chlorphenamine

470 A 12 year old boy is brought in by his mother who is concerned about the appearance of his central and lateral maxillary
incisors. She says they look like they might be decayed. They are a new family to the practice having recently moved to the area
from Birmingham. On examination you notice a generalised whitening of the incisors which seems to worse at the incisal edges.
After you have dried the teeth you notice that the whiteness appears to be linear running from mesial to distal. There is a similar
pattern on the lower incisors.
What is the most likely explanation for the presentation of the boy's incisors?
A. Tetracycline staining
B. Early caries (white spots)
C. Fluorosis
D. Demarcated Enamel Opacities
E. Previous treatment with composites

471 A 23 year old patient attends complaining of pain in an upper right molar and is keen to keep the tooth. The pain is typically
sharp in nature, is triggered by cold and persists after removal of the cold stimulus. The tooth is not tender to percussion; a
radiograph of the upper right first molar shows a large radiolucency extending to the pulp horn but no peri-radicular changes.
What treatment is most likely needed in this case?
A. Oral hygiene instruction and fluoride application
B. Excavation of caries and placement of a permanent restoration
C. Root-canal treatment
D. Indirect pulp cap and restoration
E. Direct pulp cap and restoration

472 A child of 5 years attends with pain from a grossly decayed lower right D which has a discharging sinus. He is a hemophiliac.
Which of the following is the most appropriate treatment to relieve his pain?
A. Antibiotics
B. Extraction
C. Fluoride application
D. Non-vital pulpotomy
E. Vital pulpotomy

473 You are designing a partial denture for a patient with several missing teeth in the maxilla.
The reason for surveying the model prior to designing the denture is to:
A. Measure and mark out hard and soft tissue undercuts on the casts
B. Relate the intended position of the inter-papillary plane of the patient to the casts
C. Establish the position of the post dam
D. Relate the maxillary and mandibular casts
E. Aid setting up the prosthetic teeth prior to trial insertion

474 The parotid gland is one of the major salivary glands that supply the oral cavity.
Where is the orifice of the duct of the parotid gland located?
A. At the hamular notch
B. In proximity to the incisive papilla
C. On the buccal mucosa near the maxillary second molar
D. Slightly posterior to the mandibular central incisors
E. Distal to the maxillary third molars on the palatal side

475 Any patient receiving treatment under IV sedation must have their blood pressure recorded as part of their assessment.
What is the maximum blood pressure that is generally regarded as being compatible with safe sedation in general dental practice?
A.160/95
B.140/95
C. 160/90
D.170/100
E.120/80

476 A 13 year old boy with Down's syndrome attends for the first time. He is cooperative and has no relevant medical history. He
is caries free, apart from two small occlusal cavities in his lower second primary molars. His mother requests that these teeth are
restored.

Which of the following is the most appropriate management?


A. Arranging to extract these teeth at a future visit.
B. Arranging to place two simple restorations at a future visit.
C. Reassurance that these teeth can be left until they exfoliate naturally, with no treatment being necessary.
D. Recommending use of fluoride mouthwash.
E. Take OPT to check on unerupted teeth.

477 A 3 year old child attends, showing evidence of bottle caries and also decay affecting primary molars. The child's mother has
heard that some sugars are damaging her child's teeth. She is not sure which foods to avoid.
From the following, which are you most likely to suggest she avoids?
A. Bread
B. Cheese
C. Ground almonds
D. Pure fruit juices
E. Whole fruits and vegetables

478 An adult patient complains of her prominent upper front teeth and receding chin. She says she did not get 'braces' when she
was younger because the family had to move area a lot. Her oral care and health is good, and she has a 9 mm overjet.
Where is the most suitable place to refer her? "
A. A GDP friend who has a special interest in orthodontics and has been trained to use fixed appliances.
B. A specialist practitioner who uses a lot of functional appliances.
C. A private specialist practitioner, because she's too old to get NHS treatment now.
D. A hospital consultant, as it is likely she will need surgery now to correct her problem.
E. The nearest dental hospital, although it is 70 miles away.

479 You are meeting a three year old patient for the first time. His mother explains that he had a unilateral cleft lip and palate
that was repaired in infancy. She has heard that orthodontics will usually be required when he is older. She asks you what is the
commonest orthodontic problem that occurs with a repaired cleft palate. What.:.is the most appropriate answer?
A.A contracted maxillary arch -
B. An anterior open bite.
C. An elongated maxillary arch.
D. An expanded maxillary arch.
E. An increased overjet

480 Chronic periodontitis is a disease of the periodontium initiated and sustained by microbial plaque.
Which of the following is the main diagnostic feature of the disease?
A. Tooth Mobility
B. Loss of attachment
C. Pain
D. Gingival recession
E. Drifting

481 Gingivitis and periodontitis are associated with the loss of normal tissue.
Which of the following is the principal cause of the tissue loss?
A. Bacterial enzymes such as collagenase and hyaluronidase
B. Release of lysosomal enzymes and oxygen free radicals by PMNs and macro phages
C. Cytotoxic metabolic by products such as ammonia, hydrogen sulphide and toxic amides
D. Complement activation
E. Release of bacterial endotoxins and exotoxins.

482 Localised gingival recession is recession of the gingiva confined to one surface of the tooth.
Which of the following is the most important factor leading to the development of localised gingival recession?
A. Habit activity.
B. Class 2 division 2 malocclusion with complete overbite
C. Excessive toothbrushing
D. Bony dehiscence
E. Fenestration

483 A 48 year old woman complains of a sore area on the right buccal mucosa adjacent to a restored tooth. The lesion has a
lichenoid appearance and this is confirmed histopathologically following a biopsy.
Which of the following restorative materials is most frequently associated with lichenoid changes.?
A. Gold
B. Amalgam
C. Porcelain

D. composite
E. Glass ionomer cement

484 A patient attends your surgery complaining of severe pain, swelling and mobility associated with a lower first molar tooth in
which there is a broken filling. A periapical radiograph indicates that the tooth has not been root filled and there is loss of apical
lamina dura associated with the distal root and at the bifurcation. The periodontal bone support is good. There is no significant
pocketing. What is the likely diagnosis?
A. Chronic periapical periodontitis
B. An acute periodontal-endodontic periodontitis
C. A chronic periodontal-endodontic periodontitis
D. An acute periapical periodontitis
E. Para radicular periodontitis

485 An upper incisor in a 16 year old patient has suffered trauma and the coronal tissue has been lost. The tooth has been
endodontically treated. How will the teeth best restored?
A. With a fibre post, direct core and crown.
B. With a direct core and crown.
C. With composite
D. With an indirect post-core from a pre-fabricated pattern and a crown
E. With a custom indirect post-core and crown

486 A 55 year old female presents at your surgery complaining of a sharp pain of no more than a few minutes duration arising
from her lower incisors. The pain only occurs when she eats or drinks cold or sweet foods and only lasts for as long as the stimulus
is present. A previous dentist applied a varnish to the teeth affected which seemed to help. The teeth indicated by the patient
appear healthy but when you blow air onto them the patient experiences the pain.
What is the most likely cause of the patient's pain?
A. Acute/reversible pulpitis
B. Dentine sensitivity
C. Chronic/irreversible pulpitis
D. Atypical odontalgia
E. Trigeminal neuralgia

487 As a newly qualified dentist you are offered a contract in which remuneration is on capitation basis.
What is the most important and well-recognised feature of this method of remuneration?
A. Tends to encourage under-prescribing
B. Tends to encourage high technical quality of work
C. Tends to encourage high output of procedures
D. Tends to encourage over-prescribing
E. Tends to encourage low technical quality of work

488 A 50 year old male patient has a Class III jaw relationship with an anterior open bite. It is planned to restore his lower right
second molar, which has suffered tooth wear and fracture, with an indirect restoration. This tooth has approximately 2mm of
coronal height. What would be the most suitable approach to restore this tooth?
A. Provide an adhesively retained gold onlay
B. Provide a conventional full crown
C. Increase the vertical dimension and provide a full crown
D. Surgically crown lengthen and provide a gold crown
E. Provide an adhesively retained ceramic onlay

489 Gracey curettes are specially designed for subgingival debridement and are site specific.
Which curette is recommended for use on the lingual surface of a lower second molar?
A. Gracey Curette number 1 and 2.
B. Gracey Currette number 3 and 4
C. Gracey Currette number 7 and 8.
D. Gracey Currette number 11 and 12.
E. Gracey Currette number 13 and 14.

490 Lignocaine (2%) is widely used in dental procedures. It is most often used in combination with epinephrine (1 in 80,000).
In which one of the following patients is the use of epinephrine containing local analgesia potentially hazardous?
A. Patient with severe hypertension
B. Patient on tricyclic antidepressants
C. Patient who is an alcoholic
D. Patient on monoamine oxidase inhibitors (MAOls)
E. Patient with Grand Mal epilepsy

491 An 80 year old male presents at your surgery complaining of a sharp stabbing pain of no more than 2-3 minutes duration
arising from his upper left pre-molar region. The pain can be brought on by cold stimuli but also occurs spontaneously and has
been sufficiently severe to wake the patient from sleep. He has tried taking paracetemol but this has been of no benefit. On
examination the patient has a heavily restored upper left 4, which is vital to electrical pulp testing and shows no radiographic
evidence of caries. Blowing cold air onto the tooth produces the pain but the pain also occurs spontaneously when you are
examining the patient. There is no evidence of a crack or fracture in the tooth itself.
What is the most likely cause of the patient's pain?
A. Acute/reversible pulpitis
B. Dentine sensitivity
C. Chronic/irreversible pulpitis
D. Atypical odontalgia
E. Trigeminal neuralgia

492 A patient complains of a lower incisor which has been mobile for several months. The radiograph indicates a normal level of
bony support although the periodontal space has widened. The apical bone appears normal. The tooth is tender to pressure.
Which of the following tests and or examinations would be most likely to provide a diagnosis?
A. Masticatory muscle palpation
B. Electric pulp test
C. Occlusal examination
D. Ethyl chloride test
E. Hot gutta percha application

493 A nine year old boy presents with a class I occlusion with no crowding or overjet with a grossly carious upper left 6 which is
not suitable for restoration. The upper left 7 is very near to eruption. Upper right 6, lower right 6 and lower left 6 are sound and
fissure sealed. The oral hygiene is good. What is the most appropriate extraction pattern for this patient?
A. Extract all four 1st permanent molars
B. Extract upper left 6 and lower left 6
C. Extract upper left 6 and lower right 6
D.Extract upper right 6 and upper left 6
E. Extract upper left 6 only

494 A new patient attends your practice. At the initial examination, you carry out a Basic Periodontal Examination (BPE).
What does a BPE score of 1 signify?
A. Periodontal health
B. Probing depths greater than 3.5mm
C. Presence of overhanging restorations
D. Presence of supragingival calculus
E. Presence of bleeding on probing

495 A patient presents with a history of pain in the right pre-auricular region. There is an intermittent click during opening; when
the click is not present the patient can open to a normal range. On examination the masticatory muscles, including the lateral
pterygoid, on the RHS side are tender. What is the most likely diagnosis?
A. Bruxism
B. Myofascial pain (or Pain dysfunction syndrome)
C. Disc Displacement with Reduction
D. Osteoarthrosis
E. Disc Displacement without Reduction

496 A 23 year old male presents to your surgery. He lost his upper lateral incisors some 10 years ago in a swimming pool accident.
Since then he has been wearing a 'spoon' denture which he now feels in aesthetically unacceptable. He has sought an opinion on
dental implants but has been told that he would need bone grafting for this to be successful and he is not prepared to undergo this.
His dentition is excellent with no restorations and a Class I occlusion. He wants some advice on what the best treatment might be.
Which option would you put first on your list of possibilities?
A. Two fixed - fixed resin bonded bridges using the central and canine teeth
B. Two cantilever resin bonded bridges from the central incisors and canines.
C. Two conventional fixed - fixed bridges from the canine
D. Conventional cantilever bridges from the canines
E. Cobalt chrome partial denture

497 All the films which have been manually processed by an unsupervised new trainee dental nurse in your practice are extremely
dark. Which of the following errors could lead to a dark film?
A. Insufficient exposure
B. Films have been left in the developer for too long a period of time
C. Films have been in the fixer for too long a period of time
D. The concentration of the developer is too dilute
E. The concentration of the fixer is too dilute

Q498: your colleague is having problem as every panoramic film that he produce shows extremely wide anterior teeth which are
blurred. The film also invariably has the images of the condyles cut off from the sides of the film. The most likely cause of this?
alsoYour
498 blurred.
colleague
The filmis also
having
invariably
problemshas
asthe
every
images
panoramic
of the condyles
film that cut
he produces
off from the
shows
sides
extremely
of the film.
wide
Theanterior
most likely
teeth
cause
which
ofare
this
A. The patient is incorrectly positioned too far forward relative to the image layer
B. The patient is incorrectly positioned with the chin too low
C.The patient is incorrectly positioned too far back relative to the image layer
D. The patient is in a slumped position in the machine
E. The patients is incorrectly positioned with the head tilted in the machine

499 When an individual is exposed to radiation a certain amount of radiation is needed before clinical signs of damage to somatic
cells appear. For these effects to occur a minimum radiation dose has to be exceeded and this is known as:
A. Background radiation dose
B. Threshold dose
C. Equivalent dose
D. Absorbed dose
E. Effective dose

500 Dentists are encouraged to carry out a thorough soft tissue examination and be vigilant for any lesions that might possibly be
malignant or have malignant potential. Currently it is not recommended that dentists use a mucosal staining method for screening
the general population for oral cancer. What is the most likely reason for this advice?
A. Unnecessary intervention where there is a false positive test result
B. False reassurance where there is a false negative test result
C. Low uptake of screening by those with low risk
D. Low uptake of screening by those with high risk
E. Negative result reinforcing existing bad habits

También podría gustarte